Download as pdf or txt
Download as pdf or txt
You are on page 1of 44

WHY

BUY this BOOK ?

1. Based on latest CBSE Syllabus.

2. Chapter Test is given to check the performane of the students.

3. Contains both Objective Type Questions and Subjective Type Questions.

Objective Type Questions Subjective Type Questions

1. Competency Based Questions 1. Case Study Questions


2. Assertion Reason Questions 2. Short Answer Type-I Questions
3. Case Study MCQs 3. Short Answer Type-II Questions
4. Match the Column MCQs 4. Long Answer Type Questions

As per CBSE guidelines for Term-II examination. (This paper is only for practice
purpose, not to be confused as CBSE Sample Question Paper.)

4. 6 Practice Question Papers (Practice Question Paper 1 to Practice Question


Paper 3 contains 50 MCQs of 40 marks with duration of 90 minutes.)

5. Practice Question Paper 4 to Practice Question Paper 6 has two parts. Part A
contains MCQs and Part B contains Subjective Type Questions. Question paper
is of 40 marks with duration of 2 hours.
2
Polynomial in One Variable : An algebraic expression of the form anxn + an – 1 xn – 1 + .... + a2x2 + a1x
+ a0 is called a polynomial in one variable x. In this polynomial, a0, a1, a2, ..., an are called the coefficients
of respective terms of the polynomial. The highest exponent of the variable in it is called its degree. It is

S
said to be written in the standard form, if the terms are written in increasing or decreasing order
of the exponents.

T SE
Monomial, Binomial, Trinomial : An algebraic expression with one term is called a monomial, with
two terms is called a binomial and with three terms is called a trinomial.
Linear, Quadratic and Cubic Polynomial : Polynomial of degree 1 is called a linear polynomial,

H RI
of degree 2 is called a quadratic polynomial and that of degree 3 is called a cubic polynomial.
Zero of a Polynomial :

IG P
(i) A real number a is called a zero of a polynomial p(x), if p(a) = 0.
(ii) A non-zero constant has no zero, while every real number is a zero of the zero polynomial (0).

R R
Identities : (i) (x + y)2 = x2 + 2xy + y2
E
(ii) (x – y)2 = x2 – 2xy + y2
(iv) (x + y)3 = x3 + y3 + 3xy(x + y)
(iii) x2 – y2 = (x + y) (x – y)
(v) (x – y)3 = x3 – y3 – 3xy(x – y)
PY T
(vi) (x + y + z)2 = x2 + y2 + z2 + 2xy + 2yz + 2zx (vii) If x + y + z = 0, then x3 + y3 + z3 = 3xyz.
O N

Introduction, Polynomials in One Variable


C E
© E

COMPETENCY BASED QUESTIONS Which of the following tables correctly classifies


the given polynomials as zero, linear, quadratic and
BL

I. Multiple Choice Questions


cubic polynomials?
R 1. Consider the expression xm–1 + 3; where m is
Zero polynomial 0
a constant. What is the least integer value of m for
IA

which the given expression is a polynomial in one 3 1


Linear polynomial x  , 4x
variable? (a)
(A) 2 2
(A) 0 (B) 1 (C) 2 (D) 3 Quadratic polynomial 2 x2  3 x  5
EL

R 2. Which of these is a polynomial in one variable? Cubic polynomial x3  2 x


(A) The perimeter of a square whose side length
R

is represented by the expression x . Zero polynomial 0


(B) The area of a square whose side length is Linear polynomial 4x
represented by the expression 1 + x . (b)
(B) 3 1
Quadratic polynomial x3  2 x, x
(C) The area of a rectangle whose side lengths 2 2
are represented by the expressions 2 + x and x . 2
Cubic polynomial 2x  3x  5
(D) The perimeter of a rectangle whose side
lengths are represented by the expressions x2 + x Zero polynomial 0, 4 x
and 5 – x . 3 1
Linear polynomial x
R 3. Consider the polynomials shown : (c)
(C) 2 2
2
3 1 Quadratic polynomial 2 x  3 x  5
x3 + 2x 4x 2x2 – 3x + 5 0 x
2 2 Cubic polynomial x3  2 x
Zero polynomial 0 R 15. Which of the following is a quadratic

Linear polynomial 4x polynomial ?


2 x2  3 x  5, (A) x  2 (B) x2  2 (C) x3  2 (D) 2x  2
( d)
(D) Quadratic polynomial 3 1 R 16. Which of the following is a polynomial in
x
2 2 one variable ?
3
Cubic polynomial x  2x (A)   (B) 
m
4. Consider the expression
2
1)
A x( m  3x 2 , where (C)   (D) 
m is a constant. For what value of m, will the
17.

S
expression be a cubic polynomial? R Which of the following is a binomial in y ?
(A) – 2 (B) – 1 (C) 1 (D) 2

T SE
(A) (B)  
5. Consider the polynomial in z p(z) = z 4 3
– 2z + 3.
What is the value of the polynomial at z = –1 ? (C)  (D) 
(A) 2 (B) 3 (C) 5 (D) 6 R 18. Which of the following is a trinomial in x ?

H RI
U 6. 2
x + kx + 5 is a polynomial, where k is a (A) x3  1 (B) x3  x2  x
constant. At x = 2, the value of the polynomial is 15. (C)   (D) x3  2x

IG P
What is the value of the polynomial at x = 5 ?
(A) 3 (B) 18 (C) 35 (D) 45
R 19. Which of the following is a polynomial in x ?

R R
U 7. If x  kx  6  (x  2)(x  3) for all x, the value
2
(A)  (B) 
of k is :
(A) 1 (B) – 1
E (C) 5 (D) 3 (C)   (D) 
PY T
R 8. Degree of the polynomial (x3 – 2) (x2  11) is : R 20. The coefficient of x2 in (3x 
 x 3)  

 is :
O N

 
(A) 6 (B) 5 (C) 3 (D) 2
(A) 3 (B) 1 (C) 4 (D) 2
9.
C E

A The degree of the polynomial 2 – y2 – y3  2y7


is :
R 21. Which of the following is a quadratic
polynomial in one variable ?
© E

(A) 2 (B) 7 (C) 0 (D) 3



(A)  (B) 
R 10. Degree of zero polynomial is :
BL

(A) 0 (B) 1 (C) x2 (D) 2x2  y2

(C) any natural number (D) not defined


R 22. The coefficient of x in the product (x – 1)(1 –
2x) is :
IA

R 11. What is the coefficient of x2 in the polynomial (A) 2 (B) – 1 (C) – 2 (D) 1

  R 23. Degree of which of the following polynomials
EL

is zero :
 (A) x (B) 15 (C) y (D) x  x2
(A)  3 (B)  4 (C) (D) 0
U 24. The maximum number of terms in a
R

R 12. 8 is a polynomial of degree : polynomial of degree 10 is :


(A) 9 (B) 10 (C) 11 (D) 1
(A) 1 (B) (C) 8 (D) 0 R 25. The coefficient of x in the expansion (x  3)3

R 13. (1  3x)3 is an example of : is :


(A) 1 (B) 9 (C) 18 (D) 27
(A) monomial (B) binomial
R 26. Which of the following is a polynomial?
(C) trinomial (D) none of these

R 14. Which of the following is a cubic polynomial? (A)  (B)

(A) x3  3x2 – 4x  3 (B) x2  4x – 7
(C) 3x2  4 (D) 3(x2  x  1)
(C)  (D) 
R 27. The degree of the polynomial p(x)  (x – 7)3 Hence, this is a quadratic polynomial with degree 2.
3
– x is : R 38. (a) Write an expression which is not a
(A) 3 (B) 2 (C) 1 (D) 0 polynomial.
R 28. Which of the following is not a polynomial? (b) The degree of constant polynomial is
__________.
(A) 3 (B)  Sol. (a) An expression which is not a polynomial
(C)   (D)    is  .
(b) The degree of constant polynomial is 0.
R 29. The degree of a non-zero constant polynomial U 39. Define a binomial and a quadratic
is :
polynomial. Give one example of each.

S
(A) 1 (B) –1 (C) 0 (D) –2
(2015-WKRP2OA)
R 30. A cubic polynomial is a polynomial with

T SE
Sol. An algebraic expression having two terms is
degree :
called a binomial.
(A) 1 (B) 3 (C) 0 (D) 2
Ans. 1. (B); 2. (D); 3. (A); 4. (D); 5. (D); 6. (D);
Example :  .

H RI
7. (C); 8. (B); 9. (B); 10. (D); 11. (C); 12. (D); 13. (D);
14. (A); 15. (B); 16. (A); 17. (A); 18. (B); 19. (C);
20. (C); 21. (C); 22. (B); 23. (B); 24. (C); 25. (D); An expression of the type ax2 + bx + c, where a, b

IG P
26. (D); 27. (B); 28. (C); 29. (C); 30. (B). and c are real numbers and a  0 is called a quadratic
polynomial in one variable x.
II. Fill in the blanks

R R
R 31. Degree of a linear polynomial is______ .
E Example : 2x2 + 5x + 9.
Ans. 1
R 32. Coefficient of x2 in the polynomial
PY T
 2
x  3 x  4 is ______.
O N

4 COMPETENCY BASED QUESTIONS



Ans. I. Multiple Choice Questions
C E

4
R 1. 8 is a polynomial of degree :
R 33. Degree of the polynomial x3 (2 – x3) is
______.
© E

(A) 1 (B) (C) 8 (D) 0


Ans. 6
(CBSE 2010-940112-B1)
BL

III. State True or False 2


R2. The coefficient of x in polynomial 5x3 – 3x2
R 34. Degree of the polynomial 2 – x2 – 3x3 + 2x7
+ 7x – 9 is
is 7. (A) –3 (B) 7 (C) 5 (D) –9
IA

Ans. True
R 35. Degree of the zero polynomial is zero.
R3. The degree of the polynomial P(x) = x2 +
3x + 2 is
Ans. False
EL

(A) 0 (B) 1 (C) 3 (D) 2


U 36. (1 + 5x)2 is an example of trinomial.
Ans. True II. Fill in the blank
SHORT ANSWER TYPE QUESTIONS - I
R

R 4. Degree of a zero polynomial is ______.


R 37. Classify the following polynomials as III. State True or False
linear, quadratic or cubic. Also, write its
degree.
R5. In the polynomial x6 – 7x4 + 5x2 – 8, the
(i) 2x + 5 – (x2 + 2x – 5) coefficient of x4 is 7.
(ii) 3x3 – x (3x2 – 5x + 2) U 6. 5x3 + 9x2 + 7 is a cubic polynomial.
Sol. (i) 2x + 5 – (x2 + 2x – 5)
= 2x + 5 – x2 – 2x + 5
= – x2 + 10.
1. (D) 2. (A)
Hence, it is a quadratic polynomial with degree 2.
(ii) 3x3 – x (3x2 – 5x + 2) 3. (D) 4. not defined
= 3x3 – 3x3 + 5x2 – 2x 5. False 6. True
= 5x2 – 2x.
Zeroes of a Polynomial

COMPETENCY BASED QUESTIONS Ans. 1. (B); 2. (A); 3. (C); 4. (D); 5. (D); 6. (D);
7. (C); 8. (B); 9. (B); 10. (C); 11. (B); 12. (C).
I. Multiple Choice Questions
II. Fill in the blanks
1. Which of these is a zero of the polynomial
R 13. Zero of the polynomial p(x) = 2x + 3
p(y) = 3y3 – 16y – 8 ?
is ______.
(A) – 8 (B) – 2 (C) 0 (D) 2
3
2. Given that m + 2, where m is a positive integer, Ans. 

S
2
is a zero of the polynomial q(x) = x2 – mx – 6. Which
R 14. Zeroes of the polynomial p(x) = x(x – 2)
of these is the value of m ?

T SE
(A) 1 (B) 2 (C) 3 (D) 4 (x – 3) are ______, ______ and ______.
3. Zero of the zero polynomial is : Ans. 0, 2, 3
(A) 0 (B) 1 U 15. If p(x) = 5x – 4x2 + 3, then p(0) is ______.

H RI
(C) any real number (D) not defined Ans. 3
4. Zero of the polynomial p(x)  cx  d is : III. State True or False
R 16. Zero of a zero polynomial is any real

IG P
(A) – d (B) – c (C) (D) 
number.

R R
5. The zero of the polynomial p(x)  2x  5 is : Ans. True
E R 17. Zeroes of a polynomial 3(x – 2) (x + 3)
(A) (B) (C) 0 (D)  are 2 and 3.
Ans. False
PY T
6. The zeroes of the polynomial p(x)  (x – 6) R 18. Zero of a polynomial ax is zero.
O N

(x – 5) are :
Ans. True
(A) –6, –5 (B) –6, 5 (C) 6, –5 (D) 6, 5
U 19. If p(x) = x2 + 3x + 2, then p(1) + p(2) + p(0)
C E

 
7. Find   for    is 18.
  Ans. False
© E

IV. Assertion-Reason Type


(A) (B) (C) (D) Following questions consist of two statements
BL

Assertion (A) and Reason (R). Answer these questions,


8. Which of the following polynomials has – 3 as selecting an appropriate option given below:
a zero ? (a) Both (A) and (R) are true and (R) is the correct
(A) (x – 3) (B) x2 – 9 (C) x2 – 3x (D) x2  3 explanation of (A).
IA

9. The value of the polynomial x2 – x – 1 at (b) Both (A) and (R) are true but (R) is not the
correct explanation of (A).
x  – 1 is :
EL

(c) (A) is true but (R) is false.


(A) – 3 (B) 1 (C) – 1 (D) 0 (d) (R) is true but (A) is false.
2
10. Zero of the polynomial p(x), where p(x)  ax, A/E 20. (A): Zero of the polynomial 3x – 2 is .
3
a 0 is :
R

2
(R): 3x – 2 = 0  x = .
3
(A) 1 (B) a (C) 0 (D) Ans. (a)
A/E 21. (A): If x =
3
is a zero of the polynomial
11. If     , then p(– 1) is : 2 2
2x + kx – 12, then the value of k is 5.
2
3 3
(A) (B) (C) (D) (R): 2    k    12  1 gives k = 5.
2  2
Ans. (c)
12. If p(x)  x 3 – x 2  x  1, then value of
A/E 22. (A): –1 must be added to the
 
is : polynomial p(x) = x2 – 4x – 6 to make 5a a zero
of the polynomial.
(A) (B) 4 (C) 0 (D) –2 (R): p(5) = 52 – 4 × 5 – 6 = –1.
Ans. (a)
V. Match the columns Sol. Putting x = 5 in x2 – 4x – 6, we get the value as
A/E 23. (5)2 – 4 × 5 – 6 = 25 – 20 – 6 = –1.
Column-1 Column-2 So, –1 must be subtracted to make it zero.
1. Zero of a zero polynomial (a) 2 Probable Error : Some students may write
2. Zeroes of a polynomial (b) –1 the answer as 1 instead of –1.
x2 – 5x + 6 are
3. Value of P(x) = 3 + x – 2x2 (c) 2 and –2
SHORT ANSWER TYPE QUESTIONS - II
at x = 1 is U 27. If f (x) = x3 – 3x2 + 3x – 4, find f (2) + f (– 2)
4. Zeroes of polynomial (d) any real number + f (0). (2016-9NVBEAE, 8UHO3QW, LQWG4W9)
x2 – 4 are
Sol. We have :
5. What must be subtracted (e) 2 and 3

S
from x2 – 4x – 6 to make f (x) = x3 – 3x2 + 3x – 4.
So, f (2) = 23 – 3.22 + 3.2 – 4

T SE
5 a zero of this polynomial?
= 8 – 12 + 6 – 4
(A) 1 (c); 2 (e); 3 (d); 4 (a); 5 (b) = 14 – 16.
(B) 1 (d); 2 (e); 3 (a); 4 (c); 5 (b)  f (2) = – 2. ...(i)
(C) 1 (e); 2 (d); 3 (b); 4 (c); 5 (a)

H RI
Further, f (– 2) = (– 2)3 – 3.(– 2)2 + 3.(– 2) – 4
(D) 1 (b); 2 (e); 3 (a); 4 (c); 5 (d) = – 8 – 12 – 6 – 4.
Ans. (B) 1 (d); 2 (e); 3 (a); 4 (c);  f (– 2) = – 30. ...(ii)

IG P
5 (b) Also, f (0) = 03 – 3.02 + 3.0 – 4
SHORT ANSWER TYPE QUESTIONS - I = 0 – 0 + 0 – 4.

R R
 f (0) = – 4. ...(iii)
U 24. What is the value of k in the polynomial
E Therefore, f (2) + f (– 2) + f (0)
x2 + 8x + k, if – 1 is a zero of the polynomial ? = – 2 + (– 30) + (– 4)
(2016-QXZ1V02, RB8M4SZ, 0WK6LKN)
PY T
= – 2 – 30 – 4
Sol. Let f (x) = x2 + 8x + k. = – 36.
O N

 f (– 1) = (– 1)2 + 8(– 1) + k A/E 28. Prove that 4x4 + 12x2 + 15 has no zero.
 f (– 1) = 1 – 8 + k
(2016-AKRYDAM; 2014-GB75THK)
C E

 f (– 1) = – 7 + k.
Sol. Let p(x) = 4x4 + 12x2 + 15.
Since x = – 1 is a zero of the polynomial f (x), so
Each term of this polynomial is positive for all
f (– 1) = 0  – 7 + k = 0.
© E

non-zero values of x and also p(0) = 0 + 0 + 15, i.e.,


 k = 7.
positive.
25. Find the zeroes of the polynomial :
BL

R
Therefore, its value cannot be zero at any real
(i) p(x) = 3x – 2 (ii) p(x) = ax, a  0 value of x. Hence, it has no zero.
(2016-WDRTON)
U 29. Find P(0), P(1) and P(2) for the following
Sol. We know that the zeroes of a polynomial
IA

f (x) are given by solving the equation f (x) = 0. polynomial : P(t) = 2 + t + 2t2 – t3 (2016-WDRTON)
Therefore : Sol. We have :
P(t) = 2 + t + 2t2 – t3.
EL

(i) A zero of p(x) = 3x – 2 is given by


p(x) = 0. So, P(0) = 2 + 0 + 2 × (0)2 – (0)3 = 2,
 3x – 2 = 0 P(1) = 2 + 1 + 2 × (1)2 – (1)3
 3x = 2 =2+1+2–1=4
R

and P(2) = 2 + 2 + 2 × (2)2 – (2)3 = 4.


 x= .
Thus, x = is a zero of p(x) = 3x – 2.
LONG ANSWER TYPE QUESTION
U 30. If – 2 is a zero of the polynomial
(ii) Zero of p(x) = ax is given by
and is also the zero of the polynomial px2 + kx +
p(x) = 0.
 ax = 0 2 , then find the value of k.
 x = 0. (2016-QXZ1V02, PQ58H9E)
Hence, x = 0 is the zero of p(x).
A/E 26.
What should be subtracted from the From the zero of the first polynomial, value of p
will be found and this value of p will be be used
polynomial x2 – 4x – 6, so that 5 is a zero of
in the other polynomial px 2 + kx + to find
the polynomial ? (2016-SAHSAT;
the value of k.
2015-L78MJIK, OTT6CUG, TC57UAD)
Sol. Let f (x) =  . questions, selecting an appropriate option given
below:
Then, f (– 2) =   (a) Both (A) and (R) are true and (R) is the
Since (– 2) is a zero of the polynomial f (x), so correct explanation of (A).
f (– 2) = 0. (b) Both (A) and (R) are true but (R) is not the
correct explanation of (A).
   =0 (c) (A) is true but (R) is false.
 (– 2 + p) = 0 (d) (R) is true but (A) is false.
 p = 2. ...(1) A/E 7. (A): 4x4 + 16x2 + 27 has no zero.
2
Again, let g(x) = px + kx + 2. Then, (R): Each term of this polynomial is positive

S
g(– 2) = p.(– 2)2 + k.(– 2) + 2 for all values of x. So, its value will never be zero
for any real value of x.

T SE
= 4p – 2k + 2.
A/E 8. (A): The value of P(x) = 2 + x + 2x2 – x3 at
 g(– 2) = 4.2 – 2k + 2 [From (1)]
x = 0 is 2.
 g(– 2) = 8 – 2k + 2 (R): Coeffiecient of x2 in the polynomial is 2.

H RI
 g(– 2) = 10 – 2k.
Since (– 2) is a zero of the polynomial g(x), so
SHORT ANSWER TYPE QUESTIONS - I
g(– 2) = 0. U 9. If x = – is a zero of the polynomial p(x)

IG P
 10 – 2k = 0. = 8x3 – k, then find the value of k.

R R
 2k = 10 (2016-4YWTXY8, 8JHKR2Y, LQWG4W9)
 k = 5.
E U 10. If x = is a zero of the polynomial
2x2 + kx – 12, then find the value of k.
PY T
(2016-2N5WERY, IH7W8Y5)
OBJECTIVE TYPE QUESTIONS
O N

A/E 11. What should be subtracted from the


I. Multiple Choice Questions polynomial x2 – 16x + 28, so that 1 is a zero of
C E

R 1. Zero of the polynomial P(x) = 3x – 2 : the polynomial ? (2015-47TTMHC, YYH8ND8)


(A) –3 (B) –2
U 12. If p(x) = x2 + 3x + 2, then find the value of
p(1) + p(2) + p(0).
© E

2 3
(C) (D) (2016-NALMAD; 2014-0MR812W)
3 2
BL

U 2
2. If P(y) = y – y + 1, then P(0) = SHORT ANSWER TYPE QUESTION - II
(A) –1 (B) 1 U 13. If f (x) = 5x2 – 4x + 5, find f (1) + f (– 1) +
(C) 2 (D) –2 f (0). (2016-BC1GMYC, GTZP89G)
IA

R 3. The zeroes of the polynomial


LONG ANSWER TYPE QUESTION
p(x)  (x – 6) (x – 5) are :
EL

(A) –6, –5 (B) –6, 5 (C) 6, –5 (D) 6, 5


E 14. Find the values of the polynomial p(x) =
(CBSE 2011-460017)    at x = 1,  and –2.
II. Fill in the blanks
R

(2016-5WIR5T7, FVT8MJQ)
R 4. A cubic polynomial has atmost ______
zeroes.
U 5. If p(x) = 5x2 – 4x + 5, then value of p(1) + 1. (C) 2. (B)
p(–1) + p(0) is ______ . 3. (D) 4. 3
III. State True or False 5. 25 6. True
7. (a) 8. (b)
A 6. If –1 is a zero of the polynomial x2 + 8x + k,
9. k = –1 10. k = 5
then value of k is 7.
11. 13 12. 20 13. 25
IV. Assertion-Reason Type
Following questions consist of two statements      
14. p(1) = – 1,     and p(– 2) = 59
Assertion (A) and Reason (R). Answer these    
Factorisation of Polynomials

COMPETENCY BASED QUESTIONS 8. The zeroes of the polynomial p(x) = x2 – (2k +


I. Multiple Choice Questions 1)x + 16 are positive integers. Given that k is an
integer, which of these is equivalent to the
1. Which of these is a factor of the polynomial polynomial?
p(x) = x3 + 4x + 5 ? (A) (x – 1)(x + 16) (B) (x – 1)(x – 16)
(A) (x – 1) (B) (x + 1) (C) (x – 2)(x – 8) (D) (x – 4)(x – 4)
(C) (x – 2) (D) (x + 2) 9. If (x – 1) is a factor of p(x)  x2  x  k, then

S
2. The polynomial (x – a), where a > 0, is a factor value of k is :

T SE
of the polynomial q(x) = 4 2 x2  2. Which of these (A) 3 (B) 2 (C) – 2 (D) 1
 1 10. In which of the following, (x  2) is a factor ?
is a polynomial whose factor is  x   ?
 a
(A) 4x3 – 13x  6 (B) x3  x2  x  4

H RI
(A) x2 + x + 6 (B) x2 – 5x + 4 (C) 4x3  13x – 25 (D) – 2x3  x2 – x – 19
(C) x2 + 4x – 3 (D) x2 + x – 6 11. One of the factors of (x – 1) – (x2 – 1) is :

IG P
3. The polynomial (x – a) is a factor of the (A) x2 – 1 (B) x  1 (C) x – 1 (D) x  4
4 2

R R
polynomial x – 2x + kx + k, where k is a constant. 12. The value of p for which x  p is a factor of
Which of these is the correct relation between a and
E x2  px  3 – p is :
k?
(A) 1 (B) – 1 (C) 3 (D) – 3
a2 (2  a2 ) a2 (2  a2 )
PY T
(A) k  (B) k  Ans. 1. (B); 2. (D); 3. (A); 4. (B); 5. (C); 6. (A);
1 a 1 a
7. (A); 8. (B); 9. (C); 10. (A); 11. (C); 12. (C).
O N

a2 (2  a2 ) a2 (2  a2 )
(C) k  (D) k  II. Fill in the blanks
1a 1a
C E

4. The polynomial (4x – 3)is a factor of the U 13. If(x – 1) is a factor of x2 + x + k, then
polynomial q(x) = 4x3 + x2 – 11x + 2r. What is the value of k is ______.
© E

value of r ? Ans. –2
(A) 2 (B) 3 (C) 4 (D) 11 U 14.If (x + 2) is a factor of kx3 – 13x + 6,
BL

5. The polynomial p(x) = x3 – 5x2 – x + 5 is such then value of k is ______.


that p(1) = 0 and p(– 1) = 0. Which of these is Ans. 4
equivalent to p(x) ? U 15. If (x – 1) is a factor of x3 – 13x2 + kx –
IA

(A) (x – 1)(x + 5) (B) (x – 1)(x + 1)(x + 5) 20, then value of k is ______.


(C) (x – 1)(x + 1)(x – 5) (D) (x + 1)(x – 5) Ans. 32
EL

6. A polynomial p(x) of degree n is such that p(a) III. State True or False
= 0 and p(–b) = 0. Which of the following is the
factored form of the polynomial?
U 16. (x – 3) is a factor of 2x3 – 5x2 + 2x – 18.
R

(A) (x – a)(x + b) g(x); where g(x) is a polynomial Ans. False


of degree n – 2. U 17. (x – 3) is a factor of 2x2 – x – 15.
(B) (x – a)(x + b) g(x); where g(x) is a polynomial Ans. True
of degree n
(C) (x + a)(x + b) g(x); where g(x) is a polynomial
U 18. (x + 1) is a factor of x140 + 2x141 + 2.
of degree n – 2. Ans. False
(D) (x + a)(x + b) g(x); where g(x) is a polynomial IV. Assertion Reason Type
of degree n.
Following questions consist of two statements
7. Which of these is obtained by factorizing the Assertion (A) and Reason (R). Answer these questions,
polynomial 10x2 – 9x + 2 ? selecting an appropriate option given below:
(A) (2x – 1)(5x – 2) (B) (2x – 1)(5x + 2) (a) Both (A) and (R) are true and (R) is the correct
(C) (2x + 1)(5x + 2) (D) (2x + 1)(5x – 2) explanation of (A).
(b) Both (A) and (R) are true but (R) is not the   
correct explanation of (A). =  
(c) (A) is true but (R) is false.
 (3x – 2) is a factor of f (x).
(d) (R) is true but (A) is false.
A 19. (A): (x – a) is a factor of p(x), if p(a) = 0. U 24. Find the value of k, if (x – 3) is a factor
of p(x) = 2x3 – 5x2 + 3x + k.
(R): Factor theorem.
(2016-L5Z5AX3, 3A9E2QY)
Ans. (a)
Sol. We have :
A/E 20. (A): (x – 3) is not a factor of 2x3 – 5x2 + p(x) = 2x3 – 5x2 + 3x + k.
2x – 18. So, (x – 3) is a factor of p(x).
(R) : 2(3)3 – 5(3)2 + 2(3) – 18 = 0.  p(3) = 0.

S
Ans. (c)  2(3)3 – 5(3)2 + 3(3) + k = 0
21. (A): (x – 1) is a factor of x2 + x – 2.

T SE
U  54 – 45 + 9 + k = 0
(R): Coefficient of x is 1.  63 – 45 + k = 0
Ans. (b)  18 + k = 0
 k = – 18.
SHORT ANSWER TYPE QUESTIONS - I

H RI
U 25. Find the value of ‘a’ for which (x – 1) is
U 22. Find the value of m, if x – is a factor a factor of the polynomial a2x3 – 4ax + 4a – 1.

IG P
of f (x) = x3 – 3x2 – 2mx + 1. (2016-45YEUZZ, OY1B2AV; 2015-50QINI4, FGYE8DO)
(2016-1HUU1XM, D88XL26) Sol. (x – 1) is a factor of a2x3 – 4ax + 4a – 1.

R R
3 2
Sol. We have : f (x) = x – 3x – 2mx + 1. So, a2 × 13 – 4a × 1 + 4a – 1 = 0.
E  a2 – 1 = 0  a2 = 1.
   a = ± 1.
Since    is a factor of f (x), so
 
26. Find the value of a, if (x – 1) is a factor of
PY T
U
  = 0.
  (2016-7YAIHMD, WDRTON,
O N

  L1I5XFF; 2015-BVMSAT)
  Sol. (x – 1) is a factor of  
     
C E

  So,     = 0.
       a=   .
         = 0.
© E

U 27. Find the value of k, if x – 3 is a factor of


     
p(x) = kx2 – x – 15.
BL

    =0 (2016-DZRJ8AQ, FVT8MJQ, 366QEDF; 2015-J8AN7MO)


Sol. x – 3 is a factor of kx2 – x – 15.
 m=   So, k(3)2 – 3 – 15 = 0.
IA

 9k – 18 = 0  9k = 18.
 
 m=  k=  .
EL

 m= . SHORT ANSWER TYPE QUESTIONS - II


U 23. Determine whether 3x – 2 is a factor of A 28. Show that x – 1 is a factor of the polyno-
R

3x3 + x2 – 20x + 12. (2016-J5WFNQ9, OG40F9O) mial x3 – 13x2 + 32x – 20. Hence, factorise the
Sol. Let f (x) = 3x3 + x2 – 20x + 12. polynomial. (2015-8GDMGWT, PZN9YJB)

By factor theorem, (3x – 2) will be a factor of (x – 1) is a factor will be shown using factor
    theorem. Then, the given polynomial will be divided
f (x), if    .      
    by (x – 1) and the obtained quotient will be
We have : factorised by splitting the middle term.

  =       Sol. Putting x = 1 in x3 – 13x2 + 32x – 20, we get


         .
        (1)3 – 13 × (1)2 + 32 (1) – 20 = 1 – 13 + 32 – 20 = 0.
So, (x – 1) is a factor of the given polynomial.
=    =  
Now, we divide x3 – 13x2 + 32x – 20 by x – 1 as
shown on next page :
x2 – 12x + 20 We have :
x–1 ) x3
– 13x2 + 32x – 20 ( f (–1) = (–1)3 – 7 (– 1) – 6
 f (–1) = – 1 + 7 – 6 = 7 – 7 = 0.
x – x2
3
– + So, (x + 1) is a factor of f (x).
– 12x2 + 32x – 20 Now, x3 – 7x – 6 = x3 + x2 – x2 – x – 6x – 6
= x2 (x + 1) – x (x + 1) – 6 (x + 1)
– 12x2 + 12x
+ _ = (x + 1) (x2 – x – 6)
20x – 20 = (x + 1) (x2 + 2x – 3x – 6)
20x – 20 = (x + 1) [x (x + 2) – 3 (x + 2)]
+ + = (x + 1) (x + 2) (x – 3).
0 A/E 31. Using long division method, show that

S
So, we have : the polynomial p(x) = x3 + 1 is divisible by q(x)
x3 – 13x2 + 32x – 20 = x + 1. Verify your result using factor theorem.

T SE
= (x – 1) (x2 – 12x + 20) (2016-6X1GR94, AB8ODWT)
= (x – 1) (x2 – 10x – 2x + 20) Sol. We have : p(x) = x3 + 1
= (x – 1) {x (x – 10) – 2 (x – 10)} and q(x) = x + 1.

H RI
= (x – 1) (x – 10) (x – 2). 2
x –x+1
Alternate Method :
Hence, x+1 x3 + 1
x3 – 13x2 + 32x – 20

IG P
x3 + x2
=      – –

R R
= x2 (x – 1) – 12x (x – 1) + 20 (x – 1) – x2 + 1
= (x – 1) (x2 – 12x + 20)
E – x2 – x
= (x – 1) (x2 – 10x – 2x + 20) + +
= (x – 1) {x (x – 10) – 2 (x – 10)} +x+1
PY T
= (x – 1) (x – 10) (x – 2). +x+1
– –
29. Using factor theorem, show that a + b,
O N

A/E 0
b + c and c + a are factors of (a + b + c) 3 –
Therefore, quotient = x 2 – x + 1 and
C E

(a3 + b3 + c3). (2016-AMIMAN, VIVMUN, N691PFI,


remainder = 0.
S16MA10; 2015-JNUUHUG; 2014-HY8Y7DF, JZHKVBL;
Hence, p(x) is divisible by q(x).
© E

2013-ULHRL9G)
Verification :
For showing (a + b) a factor, a = – b will be put in By factor theorem, required remainder is equal
BL

the given expression; for showing (b + c) a factor, to p(–1) and it must be equal to 0.
b = –c will be put, etc. Now, p(x) = x3 + 1
Sol. Let f (a, b, c) = (a + b + c)3 – (a3 + b3 + c3).  p(–1) = (–1)3 + 1
IA

If a = – b, then =–1+1=0
f (a, b, c) = (– b + b + c)3 – [(– b)3 + b3 + c3]  p(–1) = 0.
EL

= c3 – c3 = 0. Hence, the remainder is equal to 0.


 a + b is a factor of f (a, b, c).  p(x) is divisible by q(x).
If b = – c, then A/E 32. If 2x3 + ax2 + bx – 6 has (x – 1) as a factor
R

f (a, b, c) = (a – c + c)3 – [a3 + (– c)3 + c3]


and leaves a remainder 2 when divided by
= a3 – a3 = 0.
(x – 2), find the relations between ‘a’ and ‘b’.
 b + c is a factor of f (a, b, c).
(2016-7VJIAJM, 45YEUZZ)
If c = – a, then
f (a, b, c) = (a + b – a)3 – [a3 + b3 + (– a)3]
Using the two given conditions, two relations
= b3 – b3 = 0. between a and b shall be obtained.
 c + a is a factor of f (a, b, c).
Sol. Let f (x) = 2x3 + ax2 + bx – 6.
LONG ANSWER TYPE QUESTIONS Since (x – 1) is a factor of f (x), so
U 30. Factorise : x3 – 7x – 6 (2016-0BIOW7V) by factor theorem, we get
3
Sol. Let f (x) = x – 7x – 6 be the given 2.13 + a.12 + b.1 – 6 = 0.
polynomial. The factors of the constant term 6 in  2+a+b–6=0
this polynomials are ± 1, ± 2, ± 3, ± 6.  a+b=4 ...(1)
When f (x) is divided by (x – 2), it leaves a  – 6 = k(– 1) (– 2) (– 3)
remainder 2.  – 6 = – 6k
So, 2.23 + a.22 + b.2 – 6 = 2  k = 1.
 16 + 4a + 2b – 6 = 2 Putting k = 1 in f (x) = k(x – 1) (x – 2) (x – 3),
 4a + 2b = 2 – 10 we get
f (x) = (x – 1) (x – 2) (x – 3)
 4a + 2b = – 8
Hence, x3 – 6x2 + 11x – 6 = (x – 1) (x – 2) (x – 3)
 2a + b = – 4 ...(2)
Alternate Method :
Hence, relations between a and b are a + b = 4
and 2a + b = – 4. Let f (x) = x3 – 6x2 + 11x – 6
Putting x = 1 in f (x), we get
U 33. Factorise 9z3 – 27z2 – 100z + 300, if 3z + 10 f (x) = 13 – 6.12 + 11.1 – 6

S
is one of the factors. (2016-9NVBEAE) = 1 – 6 + 11 – 6
3 2
Sol. Let f (z) = 9z – 27z – 100z + 300.

T SE
= 0.
Let us divide 9z3 – 27z2 – 100z + 300 by 3z + 10  (x – 1) is a factor of f (x).
to get the other factors. Now, f (x) = x3 – 6x2 + 11x – 6
= x3 – x2 – 5x2 + 5x + 6x – 6
3z2 – 19z + 30

H RI
 f (x) = x2 (x – 1) – 5x (x – 1) + 6 (x – 1)
3z + 10 9z3 – 27z2 – 100z + 300  f (x) = (x – 1) (x2 – 5x + 6)
9z3 + 30z2 f (x) = (x – 1) (x2 – 2x – 3x + 6)

IG P

– –
 f (x) = (x – 1) [x (x – 2) – 3 (x – 2)]
– 57z2 – 100z

R R
 f (x) = (x – 1) [(x – 2) (x – 3)]
– 57z2 – 190z
E  f (x) = (x – 1) (x – 2) (x – 3).
+ +
Hence, x3 – 6x2 + 11x – 6 = (x – 1) (x – 2) (x – 3).
+ 90z + 300
35. Using factor theorem, show that (m – n),
PY T
A/E
+ 90z + 300
– – (n – p) and (p – m) are factors of m (n2 – p 2) +
O N

0 n (p2–m2)+p(m2–n2). (2016-8UHO3QW, HDKHAM6;


f (z) = (3z + 10) (3z2 – 19z + 30).
C E

So, 2015-7SR32MM, PDDWJZ9; 2014-HC9FWQL)


 f (z) = (3z + 10) (3z2 – 10z – 9z + 30)
 f (z) = (3z + 10) [z (3z – 10) – 3 (3z – 10)] For showing (m–n) a factor, m = n will be put in
© E

 f (z) = (3z + 10) [(z – 3) (3z – 10)]. given expression, for shown (n–p) a factor, n = p
 f (z) = (3z + 10) (z – 3) (3z – 10). will be put in the expression etc.
BL

Hence, 9z3 – 27z2 – 100z + 300 Sol. Let f (m, n, p) = m(n2 – p2) + n(p2 – m2) +
= (3z + 10) (z – 3) (3z – 10). p(m2 – n2).
34. Factorise : x3 – 6x2 + 11x – 6 If m = n, then
IA

(2016-OTMKTKH, NFYC2NF, QXZ1V02)


f (m, n, p) = n(n2 – p2) + n(p2 – n2) + p(n2 – n2)
= n(n2 – p2) – n(n2 – p2) + p × 0
Sol. Let f (x) = x3 – 6x2 + 11x – 6.
EL

= 0.
The constant term in f (x) is equal to – 6 and
 m – n is a factor of f (m, n, p).
factors of – 6 are : ± 1, ± 2, ± 3, ± 6.
If n = p, then
Putting x = 1 in f (x), we have : f (m, n, p) = m(p2 – p2) + p(p2 – m2) + p(m2 – p2)
R

f (1) = 13 – 6.12 + 11.1 – 6


= m × 0 + p(p2 – m2) – p(p2 – m2)
= 1 – 6 + 11 – 6 = 0.
= 12 – 12  n – p is a factor of f(m, n, p).
= 0. If p = m, then
 (x – 1) is a factor of f (x). f (m, n, p) = m(n2 – m2) + n(m2 – m2) + m(m2 – n2)
Similarly, (x – 2) and (x – 3) are the factors of
= m(n2 – m2) + n × 0 – m(n2 – m2)
f (x).
= 0.
Since f (x) is a polynomial of degee 3.
So, it cannot have more than three linear factors.  p – m is a factor of f (m, n, p).
 f (x) = k(x – 1) (x – 2) (x – 3) U 36. Show by long division that x – 3 is a
 x3 – 6x2 + 11x – 6 = k(x – 1) (x – 2) (x – 3) factor of 2x4 + 3x3 – 26x2 – 5x + 6.
Putting x = 0 on both sides, we get (2016-OTMKTKH, 1HUU1XM, IKZKJIT;
 – 6 = k(0 – 1) (0 – 2) (0 – 3) 2014-5JX7AP3; 2013-8CCGYLO)
2x3 + 9x2 + x – 2  (m – p) a = n – q
 a= , (proved)

A/E 39. If x3 + mx2 – x + 6 has (x – 2) as a factor


and leaves a remainder n, when divided by
(x – 3), find the values of m and n.
(2016-N9O6S82, D88XL26;
2015-MOGHEM, 6BKZMTG, T4TACJ4)
Sol. (x – 2) is a factor of x3 + mx2 – x + 6.
So, (2)3 + m (2)2 – 2 + 6 = 0.

S
 8 + 4m + 4 = 0
 4m = – 12.

T SE

Clearly, 2x4 + 3x3 – 26x 2 – 5x + 6 is exactly  m=  .
divisible by (x – 3).
So, the polynomial is x3 – 3x2 – x + 6.

H RI
Hence, (x – 3) is a factor of 2x4 + 3x3–26x2– 5x+ 6.
A/E 37. If p(x) = x3 – 4x2 + x + 6, then show that Remainder, when it is divided by x – 3, is given by
p(3) = 0 and hence factorise p(x). (3)3 – 3(3)2 – 3 + 6 = 27 – 27 + 3 = 3.

IG P
(2016-KXZIWCH, GOVVIK, DAVRPS, DAVROH; (Using Remainder Theorem)
2014-7DKLN5I; 2013-VIPDEL) So, n = 3.

R R
Sol. p(x) = x3 – 4x2 + x + 6. Thus, m = – 3 and n = 3.
So, p(3) = 33 – 4 × 32 + 3 + 6
40.
E = 27 – 36 + 3 + 6
A/E Without actual division, prove that
x  2x – 2x2  2x – 3 is exactly divisible by
4 3
= 0.
PY T
Therefore, x – 3 is a factor of x3 – 4x2 + x + 6. x2  2x – 3. (2016-VOUHSFR; 2015-47TTMHC;
O N

Now, on division, we get : 2014-DALMD; 2012-48; 2010–940109-A1, A2, 940112-B1)

x2 + 2x – 3 shall be factored as (x + 3) (x – 1) by
C E

splitting the middle term and then each of (x + 3)


and (x – 1) shall be shown as factor, by factor
© E

theorem.
Sol. Let f (x)  x4  2x3 – 2x2  2x – 3.
BL

Now, g(x)  x2  2x – 3
 x2  3x – x – 3 = x(x + 3) – 1 (x + 3)
 (x  3) (x – 1)
IA

 (x  3) and (x – 1) are factors of g(x).


 x3 – 4x2 + x + 6 = (x – 3)(x2 – x – 2) Now, f (– 3)  (– 3)4  2 (– 3)3 – 2(– 3)2 
EL

= (x – 3)(x2 + x – 2x – 2) 2 × (– 3) – 3
 81 – 54 – 18 – 6 – 3  0.
= (x – 3)[x(x + 1) – 2(x + 1)]
 (x  3) is a factor of f (x).
= (x – 3)(x – 2)(x + 1). Also, f (1)  (1)4  2(1)3 – 2(1)2  2 × 1 – 3
R

A/E 38.
If (x + a) is a factor of each of the  1  2 – 2  2 – 3  0.
polynomials x2 + px + q and x2 + mx + n, prove  (x – 1) is a factor of f (x).
that (2016-W8GW7QS, S7FTON, JE2DSHU;
 f (x) is divisible by (x  3) and (x – 1). Hence,
f (x) is exactly divisible by g(x).
2015-3EFMTRG)
2
A 41. If (x2 – 1) is a factor of ax4 + bx3 + cx2 +
Sol. (x + a) is a factor of x + px + q. dx + e, show that a + c + e = b + d = 0.
So, (– a)2 + p (– a) + q = 0 (2016-M7SGV5S, 2TGU1BO, C6RC81U;
 a2 – ap + q = 0 ...(1) 2015-Z69M0IO)
Similarly, a2 – am + n = 0, ...(2) Sol. (x2 – 1) is a factor of ax4 + bx3 + cx2 + dx + e.
because (x + a) is a factor of x2 + mx + n.
i.e., (x + 1) and (x – 1) are factors of
Subtracting (2) from (1), we get :
– ap + am + q – n = 0. ax4 + bx3 + cx2 + dx + e.
Now, (x + 1) is a factor of ax4 + bx3 + cx2 + dx + e. U 4. If (x – 3) is a factor of 2x2 – kx – 15, then
So, a (–1)4 + b (–1)3 + c (–1)2 + d (–1) + e = 0. value of k is ______.
 a–b+c–d+e=0 III. State True or False
So, a+c+e= b+d ...(1) U 5. (x – 1) is a factor of 4x3 + 3x2 – 4x – 3.
Also, (x – 1) is a factor.
IV. Assertion-Reason Type
So, a (1)4 + b (1)3 + c (1)2 + d (1) + e = 0. Following questions consist of two statements
Hence, a + b + c + d + e = 0. Assertion (A) and Reason (R). Answer these
So, (a + c + e) + (b + d) = 0 questions, selecting an appropriate option given
 (a + c + e) + (a + c + e) = 0 [From (1)] below:
 2 (a + c + e) = 0 ...(2) (a) Both (A) and (R) are true and (R) is the

S
 (a + c + e) = 0 correct explanation of (A).

T SE
But a+c+e= b+d [From (1)] (b) Both (A) and (R) are true but (R) is not the
correct explanation of (A).
So, a+c+e= b+d=0
(c) (A) is true but (R) is false.
[From (1) and (2)] (d) (R) is true but (A) is false.

H RI
Shown U 6. (A): (x – 3) is not a factor of 2x 3 – 5x 2 +
A 42.
Factorise y 3 – 2y2 – 29y – 42, using 3x – 18.
factor theorem. (2015-61OPML2, 28V72B8) (R): Coefficient of x is 3.

IG P
Sol. Given polynomial is
SHORT ANSWER TYPE QUESTIONS - I

R R
p(y) = y3 – 2y2 – 29y – 42.
U 7. Find the value of m, if x + 2 is a factor of
Now, factors of 42 are ± 1, ± 2, ± 3, ± 6 and ± 7. 2
2x + mx + 4.
E
By trial and error, we find that y = – 2 gives
(2016-6X1GR94, 71123AP, P2N17UR)
U 8. Find the value of k, if (2x – 1) is a factor
p(– 2) = (– 2)3 – 2 (– 2)2 – 29 (– 2) – 42
PY T
of f (x) = 2x3 – kx2 – x + 2.
= – 8 – 8 + 58 – 42 = 0. (2016-ETGKH87, G65WR18)
O N

So, (y + 2) is a factor of y3 – 2y2 – 29y – 42. R 9. Write the statement of Factor theorem.
(By factor theorem) (2014-1Z4HTB3)
C E

Now, we have : U 10. Find the value of k, if x – 2 is a factor of


y3 – 2y2 – 29y – 42 p(x) = x2 + kx + 2k.
© E

(2016-W8GW7QS, Q54PFTM, RDJRAV;


= y3 + 2y2 – 4y2 – 8y – 21y – 42
2015-IRUZPDV)
= y2 (y + 2) – 4y (y + 2) – 21 (y + 2)
BL

U 11. Find the value of k, if x – 3 is a factor of


= (y + 2) (y2 – 4y – 21) p(x) = x3 – 5x2 + 3kx + 9.
= (y + 2) (y2 – 7y + 3y – 21) (2016-Y12INVQ, RXTCCHZ; 2015-MOGHEM)
= (y + 2) { y (y – 7) + 3 (y – 7)}
IA

A 12. If x – a is a factor of 2x2 – mx + na, then


= (y + 2) (y – 7) (y + 3). 
prove that 
EL

(2016-7VJIAJM; 2015-TX70FDT)

OBJECTIVE TYPE QUESTIONS SHORT ANSWER TYPE QUESTION - II


R

I. Multiple Choice Questions


A 13. If both (x + 2) and (2x + 1) are factors of
2
U 1. The value of k, if (x – 1) is a factor of ax + 2x + b, prove that a – b = 0.
4x3 + 3x2 – 4x + k is : LONG ANSWER TYPE QUESTIONS
(A) 1 (B) 2
A/E 14. If the polynomial 5x3 + 4x2 – 31x + a is
(C) 3 (D) –3
U 2. If (x – 1) is a factor of p(x) = x2 + x + k,
exactly divisible by x – 2, then find the value of
then the value of k is : a. Hence, factorise the polynomial.
(2016-EGSVY52, HDKHAM6)
(A) 3 (B) 2
(C) –2 (D) 1
First find the value of by using the fact that
II. Fill in the blanks (x – 2) is factor. Then obtain the actual polynomial
U3. If (x – 3) is a factor of 2x3 – 5x2 + kx – 18, by substituting the value of a and factorise by factor
then value of k is ______. theorem.
A/E15. If the polynomial 2x3 + ax2 + 7x – 6 is A 21. State factor theorem. Using factor
exactly divisible by 2x – 1, then find the value of theorem, factorise x3 – 3x2 – x + 3.
a. Hence, factorise the polynomial. (2016-MSMMUN, YDSY58V;
(2016-71123AP, KRYDAM) 2015-DEVY2SN, CHBCHA, LU6T8O8)
A 16. Factorise : 2x3 + 7x2 – 3x – 18 22. Show that x + 2 is a factor of the
A/E

(2016-ZAJYC1U, S4623P2) polynomial 2x3 + 4x2 – 3x – 6. Hence, factorise the


A/E17. Find the values of p and q so that (x + 1) polynomial. (2016-4YWTXY8, 5LM59ZD)
and (x – 1) are the factors of the polynomial
x4 + px3 + 2x2 – 3x + q.
(2018-KVS; 2016-ZZDRO-93, 6XIGR94, 1. (D) 2. (C)

S
DTGBV5A, HILTRV1; 3. 3 4. 1

T SE
2015-28V72B8, AMCTRQA; 2014-03J7TN8) 5. True 6. (d)
A/E18. Verify if – 3 and 4 are zeroes of the 7. m = 6 8. k = 7
polynomial 2x 3 – 3x 2 – 23x + 12. If yes, then 9. (x – a) is a factor of p(x), if p(a) = 0. Also, if (x – a) is a
factorise the polynomial. (2016-K2SHJ5P, UGISTKZ; factor of p(x), then p(a) = 0.

H RI
2015-9571BXZ, 5612G9D; 2014-1AB3VAH) 10. k = – 1 11. k = 1
14. a = 6; (x – 2) (x + 3) (5x – 1)
A/E19. If (x – 3) and    are both factors of

IG P
15. a = 9 and f (x) = (2x – 1) (x + 2) (x + 3)
 
ax2 + 5x + b, then show that a = b. 16. (x + 2) (2x – 3) (x + 3)

R R
17. p = 3, q = – 3
(2016-SATSAI; 2015-7KBADPF, 40ZL433, F2A01BZ)
E 18. Yes; (x + 3) (x – 4) (2x – 1)
A 20. Using factor theorem, find the value of ‘a’, 20. a = – 29
if 2x4 – ax3 + 4x2 – x + 2 is divisible by 2x + 1.
PY T
21. (x – 1) (x + 1) (x – 3)
(2016-J5WFNQ9, IWLOOUW, S16MA11;   
22.   
O N

2015-MOGHEM, PLIGXIM)

Algebraic Identities, Factorisation


C E
© E

COMPETENCY BASED QUESTIONS 4.


Given that 1002 = a2, which expression gives
BL

the value of the expression 103 × 108 ?


I. Multiple Choice Questions
(A) a2 + 11a + 24 (B) a2 + 24a + 11
1.Which of these identities can be used to
(C) a2 + 24a + 24 (D) a2 + 11a + 11
factorize the expression 4 x2  19 x  16 ?
IA

R 5. One of the factors of (16y2 – 1)  (1 – 4y)2 is :


(A) ( x  a)2  x 2  2a  a2
(A) (4  y) (B) (4 – y) (C) (4y  1) (D) 8y
(B) ( x  a)2  x2  2 a  a2
EL

(C) ( x  a)( x  b)  x2  (a  b) x  ab U 6. If    , the value of x3 – y3


(D) ( x  a)( x  a)  x2  a2 is :
2. The volume of a cube is given by the expression
R

(A) 1 (B) – 1 (C) (D) 0


3 3 2 2
27x + 8y + 54x y + 36xy . What is the expression for U 7. a 2 2 2
 b  c – ab – bc – ca equals :
the side length of the cube? (A) (a  b  c)2 (B) (a – b – c)2
(A) 3x + 2y (B) 3x – 2y (C) (a – b  c)2
(C) 9x – 8y (D) 9x + 8y
(D)     
3. Which of these identities can be used to find
 
the value of the expression 97 × 103 ? U 8. Product of  
 
  

    is:
     
(A) ( x  y)2  x2  2 y  y2
(A)  (B)  
(B) ( x  y)2  x2  2 y  y2
(C) ( x  y  z)2  x2  y2  z2  2 xy  2 yz  2 z
(C)  (D)  
(D) ( x  y)( x  y)  x2  y2
R 9. The factorisation of (2a – b)3  (b – 2c)3  8(c (R): Identity is: (a + b + c)2 = a2 + b2 + c2 + 2ab
– a)3 is : + 2bc + 2ca
(A) (2a – b)(b – 2c)(c – a) Ans. (d)
A/E 19. (A): x4 – y4 = (x – y) (x + y) (x2 + y2)
(B) 3(2a – b)(b – 2c)(c – a)
(C) 6(2a – b)(b – 2c)(c – a) (R): x2 + y2 = (x – y) (x + y)
(D) 2a × b × 2c Ans. (c)
V. Match the columns
R 10. If    then which one of the A/E 20.
following expression is correct : Column-1 Column-2

S
1. x3 + y3 (a) 3xyz
(A) x3  y3  z3  0 (B)   
2. x2 – y2 (b) (x – 3 2 )

T SE
(C) x  y  z  3xyz (D) x3  y3  z3  3xyz (x – 3 2 )
Ans. 1. (C); 2. (A); 3. (D); 4. (A); 5. (D); 6. (D); 3. If (x + y + z) = 0, then (c) 2000
7. (D); 8. (C); 9. (C); 10. (B). x3 + y3 + z2 is equal to

H RI
II. Fill in the blanks 4. 1052 – 952 (d) (x + y) (x – y)
x y 5. x2 – 6 2 x + 18 (e) (x + y)
A 11. If   1 , (x  0, y  0), then the value

IG P
y x (x2 – xy + y2)
of x3 – y3 is______. (A) 1 (c); 2 (e); 3 (d); 4 (a); 5 (b)

R R
Ans. 0 (B) 1 (d); 2 (e); 3 (a); 4 (c); 5 (b)
(C) 1 (e); 2 (d); 3 (a); 4 (c); 5 (b)
A
x
E
12. If x  1  3 , then value of x3  1
x3
is _____ . (D) 1 (b); 2 (e); 3 (a); 4 (c); 5 (d)
PY T
Ans. (C) 1 (e); 2 (d); 3 (a); 4 (c);
Ans. 33
5 (b)
R 13. Coefficient of x in the expansion of
O N

(3x + 4)2 is ______ . SHORT ANSWER TYPE QUESTIONS - I


C E

Ans. 24
U 21. If then find the value of
III. State True or False
© E

R 14. (a + b + c)2 = a2 + b2 + c2 + ab + bc + ca. (2016-0WK6LKN, DQ35J64)


Ans. False
BL

Sol. We have :
R 15. Value of 483 – 303 – 183 = 77760.
Ans. True = 3.

16. On factorisation x2 – 6
IA

A 2 x + 18 is equal  
  = 
to (x + 3 2 ).
Now,     
   
Ans. False
EL

  =  
So,     .
IV. Assertion-Reason Type    
 
Following questions consist of two statements
Assertion (A) and Reason (R). Answer these questions, Putting = 3, we get
R

selecting an appropriate option given below:


(a) Both (A) and (R) are true and (R) is the correct (3)3 =  
explanation of (A).
(b) Both (A) and (R) are true but (R) is not the  27 =  
correct explanation of (A).
(c) (A) is true but (R) is false.  = 27 + 9
(d) (R) is true but (A) is false.
A/E 17. (A): 4x2 – 9y2 = (2x + 3y)(2x – 3y).  = 36.
(R): 4x2 = (2x)2 and 9y2 = (3y)2.
Ans. (b) A 22. Without actually calculating the cubes,
A/E 18. (A): (x + 2y + z)2 = x2 + 2y2 + z2 + 4xy + find the value of 483 – 303 – 183.
2xz + 4yz. (2016-ZZDRO93, FVT8MJQ)
Sol. We have : U 27. Factorise : 3x5 + 1536x2y3 (2015-EVY3RZ0)
483 – 303 – 183 = 483 + (– 30)3 + (– 183).
Let a = 48, b = – 30 and c = – 18. Then, Sol. 3x5 + 1536x2y3 = 3x2 (x3 + 512y3)
a + b + c = 48 – 30 – 18 = 0. = 3x2 {x3 + (8y)3}
 a3 + b3 + c3 = 3abc. = 3x2 (x + 8y)
 483 + (– 30)3 + (– 183) = 3 × 48 × (– 30) [x2 – x × 8y + (8y)2]
× (– 18) = 77760
= 3x2(x+8y)(x2 – 8xy + 64y2).
Thus, 483 – 303 – 183 = 77760.
A 23. If (3x –
4y)3 = 27x3 – 64y3 + axy2 + bx2y,
U 28. Factorise : x2 – y2 – x – y
then find the value of a + b. (2015-0ZTL90D, PZN9YJB; 2014-H7V208T)
Sol. x – y2 – x – y = (x2 – y2) – (x + y)
2

S
(2015-GJQ0TIF, 7SR32MM)
= (x – y)(x + y) – (x + y)

T SE
LHS shall be expanded using (a – b)3 identity = (x + y)[x – y – 1].
and then coefficients of different variables will
compared on LHS and RHS. A 29. If then find the value of

Sol. We have : (3x – 4y)3

H RI
(2015-ISKS7YW)
= 27x3 – 64y3 + axy2 + bx2y
27x3 – 3 × (3x)2 (4y) + 3 (3x) (4y)2 – 64y3 Sol.  = 146.

IG P
= 27x3 – 64y3 + axy2 + bx2y
 27x – 108x2y + 144xy2 – 64y3
3
  =

R R
Now,        .
= 27x3 – 64y3 + axy2 + bx2y.
E  
Comparing the coefficients on both the sides, we = 146 – 2 = 144.
get
PY T
a = 144 and b = – 108. So,  = ± 12.
So, a + b = 144 – 108 = 36.
30.
O N

A Evaluate the following product, using


A 24. Evaluate, using a suitable identity : a suitable identity : 213 × 187 (2015-ZZDR93)
C E

(997)3 (2016-1MFC8M5, HQU7G1X, L1I5XFF, Sol. 213 × 187 = (200 + 13) (200 – 13)
BKV08FS; 2015-8QPWJPI; 2014-666YMYN) = (200)2 – (13)2
Sol. (997)3 = (1000 – 3)3 = 40000 – 169
© E

= 39831.
= (1000)3 – (3)3 – 3 × (1000)2 × 3
BL

+ 3 × 1000 × (3)2
U 31. Evaluate 185 × 185 – 15 × 15, using a
suitable identity. (2015-DAVSAT)
= 1000000000 – 27 – 9000000
Sol. 185 × 185 – 15 × 15
+ 27000
IA

= 991000000 – 27 + 27000  (185)2 – (15)2 = (185 – 15)(185 + 15)


= 991026973.  (170) (200) = 34000.
32. Factorise : 6x2 + 5x – 6
EL

U (2015-BJ0WMSA)
A 25. Simplify : Sol. 2 2
6x + 5x – 6 = 6x + 9x – 4x – 6
(2015-PSN1FWJ) = 3x (2x + 3) – 2 (2x + 3)
 
R

Sol. = (2x + 3) (3x – 2).


U 33. Without actually calculating the
 squares, find the value of (105)2 – (95)2.
=
  (2015-LIMRR4J, LU6T8O8)
 Sol. (105)2 – (95)2 = (105 + 95) (105 – 95)
=    .
 = 200 × 10 = 2000.
U 26. Factorise : A 34. Without actually calculating the cubes,
(2015-76HMM3X; 2014-CD9EZBK) find the value of (0.6)3 + (0.7)3 – (1.3)3.
Sol.         (2015-Y4JRGQG, TAMMUN)
Sol. (0.6) + (0.7) – (1.3)3
3 3
 
= (0.6)3 + (0.7)3 + (– 1.3)3
   Now, (0.6) + (0.7) + (– 1.3) = 0.
So, (0.6)3 + (0.7)3 – (1.3)3 = 3 (0.6) (0.7) (– 1.3) SHORT ANSWER TYPE QUESTIONS - II
= – 3 (0.42) × 1.3
= – 1.26 × 1.3 A 40. Prove that
= – 1.638.
U 35. If   , find  .
using suitable identity. (2016-0BIOW7V, F3IL4Q7)
(2015-AQBW0LU; 2014-CTRLFCZ, 17YKKR9; 2012-43) Sol. We have :
    
Sol. We have :   3.
   
  
=

S
So,     9.
    
   

T SE
or   9 =
  
[ a3 + b3 = (a + b) (a2 – ab + b2)]
So,   9 – 2  7.
= (0.87 + 0.13)

H RI
= 1.00 = 1, Proved.
A 36. If   then find the value of
A 41. If a + b = 12 and ab = 27, then find a3 + b3.

IG P
 
  . (2015-TY48RRP; 2014-Q7KDBCR; (2016-2N5WERY, W6M431P)
  Sol. We have : a + b = 12 and ab = 27.

R R
2010-940121-A1, A2)
Now, a + b = 12.
  E  (a + b)3 = (12)3.
Sol.       .
   = 1728.
  
PY T
   a3 + b3 + 3 × 27 × 12 = 1728
     38 – 2  36.  a3 + b3 + 972 = 1728.
O N

 
 a3 + b3 = 1728 – 972
   a3 + b3 = 756.
C E

So,      6.
  A 42. If x2 + y2 = 90 and xy = 27, then find the
A 37. Evaluate, using a suitable identity : value of x3 – y3, when x > y.
© E

(105)3 (2015-TYYZTX8, VRJ8M8O) (2016-2TGU1B0, 3S41HD1)


3 3
BL

Sol. (105) = (100 + 5)


First value of (x – y) shall be found using x2 + y2
= (100)3 + 3 (100)2 × 5 + 3 × 100
= 90 and xy = 27. Then, identity (x – y)3 = x3 – y3
× (5)2 + 53. – 3xy (x – y) will be used.
IA

= 1000000 + 3 × 10000 × 5 +
Sol. We have : x2 + y2 = 90 and xy = 27.
3 × 100 × 25 + 125 We have :
EL

= 1000000 + 150000 + 7500 + 125 (x – y)2 = x2 + y2 – 2xy


= 1157625. = 90 – 2 × 27
U 38. Factorise : 5x2 – 8x – 13 (2015-UNIHEM) = 90 – 54 = 36.
R

2 2  (x – y)2 = 36.
Sol. 5x – 8x – 13 = 5x – 13x + 5x – 13
 (x – y) = 6 [Since x > y]
= x (5x – 13) + 1 (5x – 13) Now, (x – y)3 = (6)3.
= (5x – 13) (x + 1).     = 216.
A 39. Give possible expressions for the length  x3 – y3 – 3 × 27 × 6 = 216.
and breadth of the rectangle, whose area is  x3 – y3 – 486 = 216.
6a2 + a – 12. (2016-SA16MA11; 2015-WXD7ZVQ)  x3 – y3 = 216 + 486
Sol. 6a2 + a – 12 = 6a2 + 9a – 8a – 12  x3 – y3 = 702.
= 3a (2a + 3) – 4 (2a + 3) A 43. Evaluate 923, using a suitable identity.
= (2a + 3) (3a – 4). (2016-7YAIHMD, HQU7G1X)
So, possible expressions for the length and breadth Sol. We have :
of the rectangle are (2a + 3) and (3a – 4). (92)3 = (100 – 8)3.
(92)3 (100 – 2)3 = (100)3 – 83 – 3 × 100 × 8 A 46. Using a suitable identity, evaluate
(100 – 8) (104)3. (2016-ARBRAV, FVT8MJQ)
     Sol. We have :
 = 1000000 – 512 – 2400 × 92 (104)3 = (100 + 4)3.
 = 1000000 – 512 – 220800  (104)3 = (100)3 + (4)3 + 3 × 100 × 4 × (100 + 4)
 = 778688.
     
A 44. Evaluate (501) 3 , using a suitable
 (104)3 = 1000000 + 64 + 1200 × 104.
identity. (2016-9TCIE1Y, JEZHSNU)
 (104)3 = 1000000 + 64 + 124800
Sol. We have :
(501)3 = (500 + 1)3  (104)3 = 1124864.

S
= (500)3 + (1)3 + 3 × 500 × 1 (500 + 1) A 47. If 3x + 2y = 12 and xy = 6, find the value

T SE
      of 9x2 + 4y2. (2016-DE1F4MQ, PQ58H9E)
= 125000000 + 1 + 1500 × 501 Sol. We have : 3x + 2y = 12 and xy = 6.
= 125000000 + 1 + 751500 Hence, (3x + 2y)2 = (12)2
= 125751501.

H RI
   = 144
A 45. If find the value of    = 144.

IG P
(2016-MAXMAN, 4YWTXY8, RB8M4SZ)     = 144
   = 144

R R
2 2 2
Identity (a + b) = a + b + 2ab shall be used   =
 144 – 72
twice.
E   =
 72.
Sol. We have :
48. Simplify : (2x + p – c)2 – (2x – p + c)2
PY T
A

 = 6. (2016-H5SCIQR, WYQXG6V)
O N

Sol. We know that a2 – b2 = (a + b) (a – b).


  = 62
C E

    Hence, (2x + p – c)2 – (2x – p + c)2


  = (2x + p – c + 2x – p + c) (2x + p – c – 2x + p – c).
(On squaring both sides)
(2x + p – c)2 – (2x – p + c)2 = (4x) (2p – 2c)
© E


 = 36.  (2x + p – c)2 – (2x – p + c)2 = 8x(p – c).
 
BL

A/E 49. If find the value of


   = 36.
IA

using only the positive value of


  = 36 – 2
EL

(2016-NFYC2NF, YY8420J)
  = 34.
Sol. We have :   .
 
R

2
    = (34) .
  We know that    =   .
 
(On squaring both sides)  

   =   .
   = 1156.   
 

   = 11 – 2
   = 1156.   
 
 
   
  = 1156 – 2  

   = 9.
  = 1154.   
 
   = 32    = 216
  
 
  = 3.   = 216 + 18 = 234.

  
 
(Taking square roots of both sides Taking    = – 6, we get

and taking only the positive value)  
  = (– 6)3 = – 216.
   = 33 (Cubing both sides)   
    
 

S
   
       = – 216.
     = 27.  
 

T SE
    = 27     = – 216

H RI
  = 27 + 9
   = – 216 – 18

= – 234.
  = 36.
 

IG P
A 51. Expand (– 2x + 5y – 3z)2.

R R
Hence,  = 36.
  (2016-OTMKTKH, JE2DSHU, P2N17UR)
Sol. We will use the following identity :

A 50. If
E then find the value of
(a + b + c)2 = a2 + b2 + c2 + 2ab + 2bc + 2ca
Hence, we have :
PY T
(2016-NUUGZQU, RB8MUSZ)    =    
O N

=      
Sol. We know that     
C E

  = =      .
     .
     
© E

  = =      .
     .
BL

 A 52. If x + y + z = 10 and x + y + z = 40, 2 2 2


 
  then find xy + yz + zx.
   = 38 – 2  
   



(2016-OG40F9O; 2015-ZG3KB1S)
 
IA

Sol. x + y + z = 10
   = 36. So, (x + y + z)2 = 102
  
 
EL

 x2 + y2 + z2 + 2xy + 2yz + 2zx = 100.


  =±6 (Taking square roots  40 + 2 (xy + yz + zx) = 100.
of both sides)  2 (xy + yz + zx) = 100 – 40
R

 2 (xy + yz + zx) = 60
Taking   
 we get
   xy + yz + zx =  .
  = 63 .
   A 53. Factorise : 125p 3 – 27q3 – 225p 2q +
  135pq2 (2015-BUMHEM, AQPK45K, PZ93IFN)

   = 216. Sol. 125p – 27q3 – 225p2q + 135pq2


3
    
  = (5p)3 – (3q)3 – 3 × (5p)2 × 3q + 3 × 5p × (3q)2
   = 216 = (5p – 3q)3 = (5p – 3q)(5p – 3q)(5p – 3q).
    
  A 54. Factorise : (2x + y)3 – (x – 2y)3
 = 216 (2015-1I03XZ9)
   3 3
Sol. (2x + y) – (x – 2y)
= (2x + y – x + 2y) {(2x + y)2 + (2x + y) (x – 2y) Sol. 216a3 + 343b3 + 756a2b + 882ab2
+ (x – 2y)2} = (6a)3 + (7b)3 + 3 × (6a)2 × 7b + 3 × 6a × (7b)2
     = (6a + 7b)3
= = (6a + 7b) (6a + 7b) (6a + 7b).
      
   
U 59. Find the product :
(x + y + 2z) (x2 + y2 + 4z2 – xy – 2yz – 2zx)
= (x + 3y) {7x2 – 3xy + 3y2}.
(2015-6KN4APE)
      2 2 2
Sol. (x + y + 2z) (x + y + 4z – xy – 2yz – 2zx)
A/E 55. If       then find
      = (x + y + 2z) {x2 + y2 + (2z)2– x × y – y × 2z – 2z × x)

S
the value of x. (2016-71123AP, MAXMAN) = x3 + y3 + (2z)3 – 3xy (2z)

T SE
= x3 + y3 + 8z3 – 6xyz.
 
Sol. Let    . U 60. Expand : (– 3x + 5y – 2z)2
  (2016-BBLTON, JE2DSHU, NUUGZQU;
Then, a+b+c=   =   

H RI
  2015-8QPWJP1)
2
Sol. (– 3x + 5y – 2z)
=   . = (– 3x)2 + (5y)2 + (– 2z)2 + 2 (– 3x) (5y)

IG P
 a3 + b3 + c3 = 3abc + 2 (5y) (– 2z) + 2 (– 2z) (– 3x)
= 9x2 + 25y2 + 4z2 – 30xy – 20yz + 12zx.
  

R R
     U 61. Find the product :
   E    
      (5a – 3b) (25a2 + 15ab + 9b2)
    (2016-P2N17UR, UGISTKZ, HILTLTRV1, GHSZCQU;
PY T
=   
   2015-21L1PU8, BZ27KP0)
Sol. (5a – 3b) (25a2 + 15ab + 9b2)
O N

=  . = (5a – 3b) {(5a)2 + 5a × 3b + (3b)2}


C E

= (5a)3 – (3b)3 {Using a3 – b3 = (a – b)


 = . (a2 + ab + b2)}
3 3
 x = 8. = 125a – 27b .
© E

U 56. Factorise : A 62. If a – b = 7 and a2 + b2 = 85, find a3 – b3.


BL

1000x3 + 1331y3 + 3300x2y + 3630y2x (2016-RRQOKMY, IWLOOUW, HCXDOGK, S4623P2;


2015-47TTMHC)
(2015-PDDWJZ9; 2014-DLRT39E)
Sol. a–b = 7  (a – b)2 = 49.
Sol. 1000x3 + 1331y3 + 3300x2y + 3630y2x
IA

2 2
 a + b – 2ab = 49.
= (10x)3 + (11y)3 + 3 × 10x × 11y(10x + 11y)
 85 – 2ab = 49
= (10x + 11y)3
 2ab = 85 – 49.
EL

[Using a3 + b3 + 3ab (a + b) = (a + b)3]


 2ab = 36
= (10x + 11y)(10x + 11y)(10x + 11y).
 ab = 18.
U 57.
If a + b + c = 5 and ab + bc + ca = 12, So, a3 – b3 = (a – b) (a2 + b2 + ab).
R

find a + b2 + c2.
2
(2015-6KBELZ0) = 7 (85 + 18)
Sol. a + b + c = 5. = 7 × 103 = 721.
So, (a + b + c)2 = 52. U 63. Factorise :
 a2 + b2 + c2 + 2ab + 2bc + 2ca = 25.
9a2 + 49b2 + c2 – 42ab + 14bc – 6ac
 a2 + b2 + c2 + 2 (ab + bc + ca) = 25. (2015-IRUZPDV; 2014-SKVFD44)
 a2 + b2 + c2 + 2 × 12 = 25 Sol. Given expression
 a2 + b2 + c2 = 25 – 24 = (– 3a)2 + (7b)2 + (c)2 + 2 × (– 3a)(7b)
 a2 + b2 + c2 = 1. + 2 × (7b) × (c) + 2 × (– 3a) × (c)
A 58. Factorise : So, (– 3a) + (7b)2 + (c)2 – 42ab + 14bc – 6ac
2

216a3 + 343b3 + 756a2b + 882ab2 = (– 3a + 7b + c)2


(2015-6KN4APE) = (– 3a + 7b + c)(– 3a + 7b + c).
Sol. x6 – 64 = (x2)3 – (4)3.
   
A 64. Simplify :     = (x2 – 4) [(x2)2 + x2 × 4 + (4)2]
    = (x2 – 4) (x4 + 4x2 + 16).
(2016-HCXDOGK, I5K7FNY, 366QEOF;
2015-9571BXZ, EUJPWWX, KVCV6TQ)
= (x + 2) (x – 2) {(x2 + 4)2 – 4x2}
= (x + 2) (x – 2) {(x2 + 4)2 – (2x)2}
   
Sol.     = (x + 2) (x – 2) (x2 + 4 + 2x)
    (x2 + 4 – 2x).
= [(x + y) (y – x) + 2x2]
A 70. Factorise (x – y)2 – 7 (x2 – y2) + 12 (x + y)2.
= y2 – x2 + 2x2 = y2 + x2.
(2015-YYH8ND8)

S
U 65. If a + b + c = 4 and a2 + b2 + c2 = 14, find Sol. (x – y)2 – 7 (x2 – y2) + 12 (x + y)2
ab + bc + ca. (2015-EVY3RZ0, PDDWJZ9) = (x – y)2 – 7 (x + y) (x – y) + 12 (x + y)2

T SE
Sol. (a + b + c) = 4. Let x – y = a and x + y = b.
 (a + b + c)2 = 16. So, given expression
 a2 + b2 + c2 + 2ab + 2bc + 2ca = 16 = a2 – 7ab + 12b2.

H RI
 14 + 2 (ab + bc + ca) = 16 = a2 – 4ab – 3ab + 12b2
 2 (ab + bc + ca) = 16 – 14 = 2. = a (a – 4b) – 3b (a – 4b)

IG P
 ab + bc + ca =  = (a – 4b) (a – 3b).
= {x – y – 4 (x + y)} {x – y – 3 (x + y)}
66. Factorise

R R
U x4 – y4. (2015-OTT6CUG, QHYQUU;
= (x – y – 4x – 4y) (x – y – 3x – 3y)
2013-BFMFPM7, 48B6UJF)
E = (– 3x – 5y) (– 2x – 4y).
Sol. x – y = (x ) – (y ) = (x2 – y2)(x2 + y2)
4 4 2 2 2 2
= (3x + 5y) (2x + 4y)
PY T
= (x – y)(x + y)(x2 + y2). = 2 (3x + 5y) (x + 2y).
LONG ANSWER TYPE QUESTIONS
O N

U 67. Factorise :
A 71. What are the possible expressions for
C E

(2015-GIQQLJB) the dimensions of a cuboid, whose volume =


12kx3 + 34kx2 + 10kx ?
Sol. 
© E

(2016-0WK6LKN, 6OGFTVE, 0WK6LKN)


BL

    The given expression shall be factorised into


=         
    three factors. Then, each factor will give a
dimension. Answer may not be unique.
=      .
IA

         Sol. We have :
     
Volume of the cuboid = 12kx3 + 34kx2 + 10kx.
A/E 68. Factorise 4x2 + y2 + 25z2 + 4xy – 10yz – = 2kx(6x2 + 17x + 5)
EL

20zx and hence find its value when x = – 1, y = = 2kx(6x2 + 15x + 2x + 5)


2 and z = – 3. (2015-GOVDEM)
= 2kx{3x(2x + 5)+1(2x+ 5)}
2 2 2
Sol. 4x + y + 25z + 4xy – 10yz – 20zx
R

= 2kx(2x + 5) (3x + 1).


= (2x)2 + y2 + (– 5z)2 + 2 × (2x)( y) So, length = 2kx, breadth = (2x + 5) and height
+ 2( y) (– 5z) + 2 × (– 5z)(2x) = (3x + 1).
2
= (2x + y – 5z) = (2x + y – 5z) (2x + y – 5z). Note that, the answer is not unique.
Its value at x = – 1, y = 2 and z = – 3 is U 72. Factorise a3 – 27b3 + 7a2b – 21ab2.
= {2(–1) + 2 – 5 (– 3)}2 = (– 2 + 2 + 15)2 (2016-1EDQVR7, 6X1GR94)
= (15)2 = 225. Sol. We have : a3 – 27b3 + 7a2b – 21ab2.
 (a)3 – (3b)3 + 7ab (a – 3b)
U 69. Factorise : x6 – 64 We know that
(2016-SAHSAT; 2015-TC57UAD)
a3 – b3 = (a – b) (a2 + ab + b2).
So, a3 – (3b)3 + 7ab (a – 3b)
Can also be factorised by first writing x6 – 64 = = (a – 3b) [a2 + a.3b + (3b)2]
(x3)2 – 82 = (x3 + 8) (x3 – 8), etc. + 7ab (a – 3b)
= (a – 3b) [a2 + 3ab + 9b2 + 7ab]
   =2
= (a – 3b) [a2 + 10ab + 9b2]     
 
= (a – 3b) [a2 + 9ab + ab + 9b2]
= (a – 3b) [a (a + 9b) + b (a + 9b)]     =2
= (a – 3b) [(a + b) (a + 9b)]
= (a – 3b) (a + b) (a + 9b).  
   
A 73. If a + b + c = 6, find the value of  
(2 – a)3 + (2 – b)3 + (2 – c)3 – 3(2 – a)(2 – b)(2 – c).    =2 .
(2016-2N5WERY, 6HSZCQU)

S
 =2 –3
With the help of a + b + c = 6, value of (2 – a) +

T SE
(2 – b) + (2 – c) shall be shown equal to 0 and
  =– .
then identity a3 + b3 + c3 = 3abc shall be used.
Sol. We have : U 76. Factorise 2y3 + y2 – 2y – 1.
(2 – a) + (2 – b) + (2 – c)

H RI
(2016-6LM59ZD, DNT7SPS, 3A9E2QY)
= 6 – (a + b + c)
Sol. We have : f (y) = 2y3 + y2 – 2y – 1
=6–6 (Since a + b + c = 6)
= y2 (2y + 1) – 1 (2y + 1)

IG P
= 0.
Hence, (2 – a)3 + (2 – b)3 + (2 – c)3 = (y2 – 1) (2y + 1)

R R
= 3(2 – a)(2 – b)(2 – c). = (y + 1) (y – 1) (2y + 1)
So, (2 – a)3 + (2 – b)3 + (2 – c)3
E     
– 3(2 – a)(2 – b)(2 – c) 3 2
Hence, 2y + y – 2y – 1 = (y + 1) (y – 1) (2y + 1).
= 3 (2 – a)(2 – b)(2 – c) – 3(2 – a)(2 – b)(2 – c)
PY T
= 0.
U 77. Find the value of 8a3 – 27b3 + 90ab +
O N

125, if 2a = 3b – 5. (2016-8JHKR2Y, BUB9YRQ)


A 74. Without actually calculating the
Sol. We have :
cubes, find the value of (1)3 + (2)3 + 2(4)3 +
C E

2a = 3b – 5
(– 5)3 + (– 6)3. Also, write the identity used.
or 2a – 3b = – 5.
(2016-2TGU1B0)
© E

Cubing both the sides, we get


Sol. We have :
(2a – 3b)3 = (– 5) 3 .
(1)3 + (2)3 + 2(4)3 + (– 5)3 + (– 6)3
BL

 (2a)3 – (3b)3 – 3.2a.3b(2a – 3b) = – 125


= (1)3 + (2)3 + (4)3 + (4)3 + (– 5)3 + (– 6)3
     
 (1)3 + (4)3 + (– 5)3 + (2)3 + (4)3 + (– 6)3
 8a3 – 27b3 – 18ab × (– 5) = – 125
IA

 3.1.4.(– 5) + 3.2.4.(– 6)
 8a3 – 27b3 + 90ab + 125 = 0.
(If a + b + c = 0, then a3 + b3 + c3 = 3abc)
U 78. Factorise :
EL

 – 60 – 144 = – 204.
125a + 27b3 + 225a2b + 135ab2 + 10a + 6b
3
 (1)3 + (2)3 + 2(4)3 + (– 5)3 + (– 6)3 = – 204.
(2016-71123AP, MVTHNRO)
A 75. If evaluate Sol. We have :
R

125a3 + 27b3 + 225a2b + 135ab2 + 10a + 6b.


(2016-6HSZCQU, DNT7SPS, 45YEUZZ)
=     
Sol. We have :
=   
 = .      
=   
   =( )3 . =   
  
  =    
(Cubing both the sides) A 79. What are the possible expressions for
the dimensions of a cuboid whose volume =
   =2 . 15kx2 – 5kxy – 10y2k ? (2016-DZRJ8AQ, N9O6S82)
    
  Sol. We have :
Volume of the cuboid
   
= 15kx2 – 5kxy – 10y2k. Sol.
=  
 
=         
=
=      
= 5k(x – y) (3x + 2y).
Therefore, the length of cuboid = 5k,      
=
breadth of cuboid = (x – y)   
and height of cuboid = (3x + 2y)
    

S
and many more possible expressions may be for =  .
the dimensions.  

T SE
A 80. If x and y are the two positive real A 83. Without actually calculating the cubes,
numbers such that 9x2 + 4y2 = 97 and xy = 6, find the value of (– 1)3 + (– 2)3 + (– 3)3 + (– 4)3 +
then find the value of 27x3 + 8y3. 2(5)3. Also, write the identity used.

H RI
(2016-MQVVG67, PQ58H9E, V5LM0CH) (2016-AMIMAN, GOVMVIK, OU6BHOO, S94S5OS;
2015-AMCTRQA, 5612G9D)
Sol. We know that
Sol. (– 1)3 + (– 2)3 + (– 3)3 + (– 4)3 + 2 (5)3

IG P
(3x + 2y)2 = (3x)2 + (2y)2 + 2.3x.2y
= (5)3 + (– 1)3 + (– 4)3 + (– 2)3 + (– 3)3 + (5)3
= 9x2 + 4y2 + 12xy

R R
= 3 × 5 × (– 1)(– 4) + 3 (– 2)(– 3) (5)
= 97 + 12 × 6
[Using a3 + b3 + c3 = 3abc, if a + b + c = 0]
(Putting 9x2 + 4y2 = 97 and xy = 6)
E = 60 + 90 = 150.
= 97 + 72.
PY T
 2
(3x + 2y) = 169. U 84. Simplify :    
   
O N

 2
(3x + 2y) = (13) . 2    
 (3x + 2y) = 13. (2016-QUMTMAD, CEGNIS6; 2015-8QPWJP1)
C E

 (3x + 2y)3 = (13)3 Sol.    


   
3 3    
 (3x) + (2y) + 3.3x.2y.(3x + 2y) = 2197.
© E

 27x3 + 8y3 + 18xy (3x + 2y) = 2197 =            


         .
BL

 27x3 + 8y3 + 18 × 6 × 13 = 2197           


 27x3 + 8y3 + 1404 = 2197            
             
 27x3 + 8y3 = 2197–1404            
IA

 27x3 + 8y3 = 793.


=       
EL

A 81. Factorise :
=  .
(2016-OGR0F9O, 9TCIE1Y;
85.
R

2015-21L1PU8, CSIG47O; 2014-0MR812W) A Prove that :


Sol. Given expression (x + y)3 + (y + z)3 + (z + x)3 – 3 (x + y) (y + z) (z + x)
= 2 (x3 + y3 + z3 – 3xyz)
    (2016-HDKHAM6, 1EDQVR7, S16MA11, VOUHSFR;
         
    2015-76HMM3X, BUMHEM, LIMRR4J, 0ZTL90D, AMIHEM)
Sol. LHS = (x + y)3 + (y + z)3 + (z + x)3
     
          – 3 (x + y) (y + z) (z + x)
      = (x + y + y + z + z + x) {(x + y)2
+ (y + z)2 + (z + x)2 – (x + y) (y + z)
A 82. Simplify : – (y + z) (z + x) – (z + x) (x + y)}
= 2 (x + y + z) {x2 + y2 + 2xy + y2 + z2
(2016-DQ35J64, LQWG4W9; + 2yz + z2 + x2 + 2zx – xy – zx – y2 – yz
2015-2GJMA9X, AMIHEM, JNUUHUG) – yz – xy – z2 – zx – zx – yz – x2 – xy}
= 2 (x + y + z) (x2 + y2 + z2–xy – yz – zx)  5x + 7y = 41. (Taking only the +ve value)
= 2 (x3 + y3 + z3 – 3xyz) Putting the value of 5x + 7y, 25x2 + 49y2 and xy
= RHS, Hence, proved. in (1), we get :
A 86. If ab + bc + ca = 0, find the value of 125x3 + 343y3 = 41 (841 – 35 × 12).
= 41 (841 – 420)
= 41 × 421
= 17261.
(2016-NUUGZQU, SAHSAT, Z1XF9CV, MVTHNRO;
2015-AKASVH1)
A/E 89. If find the value of

considering only the positive

S
Using ab + bc + ca = 0, values of bc, ca and ab
shall be substituted and then the expression shall

T SE
be simplified. value of (2016-OU6BHOO; 2014-1JSBHY6)

Sol.   Sol.   (Given)

H RI
=  
       
       

IG P
[Since ab + bc + ca = 0]  
    

R R
=    
     
E   
 
=  = 0. (Taking only +ve value)
   
PY T
Taking cubes on both sides, we get
A 87. If x and y are two positive real numbers
O N

such that 8x3 + 27y3 = 730 and 2x2y + 3xy2 = 15,  


then evaluate 2x + 3y. (2016-DE1F4MQ, S16MA10)    
 
C E

Sol. We have : (2x + 3y)3


= (2x)3 + (3y)3 + 3 × (2x)2 × 3y + 3 (2x) (3y)2.     
© E

= 8x3 + 27y3 + 36x2y + 54xy2


 
= 730 + 18 (2x2y + 3xy2)
BL

      
= 730 + 18 × 15  
= 730 + 270
    
= 1000 = 103.
IA

 2x + 3y = 10.
     
88. If x and y are two positive real numbers
EL

A
such that 25x2 + 49y2 = 841 and xy = 12, then
find the value of 125x3 + 343y3.       
(2016-DTGBV5A; 2015-AQPK45K, LU6T8O8;
R

2014-A20Q1Y1) A/E 90. Factorise :


3 3
Sol. 125x + 343y
= (5x)3 + (7y)3. (2016-GRFMUM; 2014-2DIM627)
2
= (5x + 7y) (25x + 49y – 5x × 7y) 2 Sol. Given expression
= (5x + 7y) (25x2 + 49y2 – 35xy) ...(1)        
Now, we have :       
2 2 2
(5x + 7y) = 25x + 49y + 2 × 5 × 7xy
       
= 25x2 + 49y2 + 70xy.
= 841 + 70 × 12  
= 841 + 840 = 1681 = (41)2.   
A 91. What are the possible expressions for Sol. Given expression
the dimensions of the cuboid whose volume = = [(x3 + y3 + 3x2y + 3xy2) – (x3 – y3 – 3x2y
6y3 + y2 – 12y + 5 ? (2016-Z1XF9CV; 2015-EUJPWWX) + 3xy2)] – 6y(x2 – y2)
Sol. 6y3 + y2 – 12y + 5. = [x3 + y3 + 3x2y + 3xy2 – x3 + y3 + 3x2y – 3xy2]
Putting y = 1, we get : – 6yx2 + 6y3
3 2 2 3 3
6(1)3 + (1)2 – 12 × 1 + 5 = 0. = 2y + 6x y – 6yx + 6y = 8y .
So, (y – 1) is a factor of 6y3 + y2 – 12y + 5. A 95. It is given that 3a + 2b = 5c, then find
Now, we have : the value of 27a3 + 8b3 – 125c3, if abc = 0.
6y3 + y2 – 12y + 5 (2016-8UHO3QW, M7SGV5S; 2015-6KN4APE;
2014-CTRLFCZ)

S
=     
3 3 3
Sol. Using identity x + y + z – 3xyz
=      .

T SE
= (x + y + z)(x2 + y2 + z2 – xy – yz – zx),
=   
= (3a)3 + (2b)3 + (–5c)3 – 3 × 3a × 2b × (–5c)
   
=     } = [(3a + 2b + (– 5c)][(3a)2 + (2b)2 + (–5c)2

H RI
=    . – (3a)(2b) – (2b)(–5c) – (–5c)(3a)]
So, possible dimensions of the cuboid are (y – 1), So, 27a + 8b – 125c3 = [3a + 2b – 5c] ×
3 3

IG P
(3y + 5) and (2y – 1). [9a2 + 4b2 + 25c2 – 6ab + 10bc + 15ac] – 90abc
 27a3 + 8b3 – 125c3 = [5c – 5c] ×

R R
A 92. Factorise :
E [9a2 + 4b2 + 25c2 – 6ab + 10bc + 15ac] – 90abc
(2016-RXTCCHZ, H5SCIQR; 2015-F30NH6Q) = 0 – 90 × 0 = 0.
A 96. Prove that :
PY T
Sol.  
(a – b2)3 + (b2 – c2)3 + (c2 – a2)3 =
2
O N

3 (a + b) (b + c) (c + a) (a – b) (b – c) (c – a)
=   
 
  
   
    (2016-6LM59ZD, OG40F9O, BBLTON, V5L9L3U;
C E

   
          2015-GIQQLJB, GJQOTIF, 47TTHHC, KVCV6TQ)
Sol. LHS = (a2 – b2)3 + (b2 – c2)3 + (c2 – a2)3
=        
© E

         = [(a2 – b2) + (b2 – c2) + (c2 – a2)]


       
[(a2 – b2)2 + (b2 – c2)2 + (c2 – a2)2
BL

 – (a2 – b2)(b2 – c2) – (b2 – c2)


      
 (c2 – a2) – (a2 – b2)(c2 – a2)] +
   3 × (a2 – b2) (b2 – c2)(c2 – a2)
IA

=            . = 0 + 3(a2 – b2)(b2 – c2)(c2 – a2).


   = 3(a – b)(a + b)(b – c)(b + c)(c – a)(c + a)
EL

U 93. Factorise : a12x4 – a4x12 = 3(a + b)(b + c)(c + a) (a – b)(b – c)(c – a).
(2016-L1I5XFF, EFUYQR, G65WR18; 2014-APAHR7) = RHS.
Hence, proved.
Sol. a12x4 – a4x12
R

Alternate Method :
= a4x4 (a8 – x8).
We have : (a2 – b2) + (b2 – c2) + (c2 – a2) = 0.
= a4x4[(a4)2 – (x4)2]
So, (a2 – b2)3 + (b2 – c2)3 + (c2 – a2)3
4 4 4 4 4 4
= a x [(a – x ) (a + x )]. = 3(a2 – b2) (b2 – c2) (c2 – a2)
= a4x4[[(a2)2 – (x2)2][a4 + x4]] [Using x3 + y3 + z3 = 3xyz, if x + y + z = 0]
= a4x4[(a2 – x2)(a2 + x2)(a4 + x4)] = 3(a + b)(a – b)(b + c) (b – c)(c + a)(c – a).
= a4x4[(a – x)(a + x)(a2 + x2)(a4 + x4)]. A/E 97. Factorise : x6 + 5x3 + 8
(2016-BUR9YRQ, DE1F4MQ; 2015-TAMMUN)
U 94. Simplify :
(x + y)3 – (x – y)3 – 6y (x + y) (x – y)
(2016-RB8M4SZ, HILLTRV1; 2015-IRUZPDV; It will be factorised using (a3 + b3 + c3 – 3abc) =
2014-AVHHDG9) (a + b + c) (a2 + b2 + c2 – ab – bc – ca).
Sol. x6 + 5x3 + 8 = x3 + y3 + 3x2y + 3y2x – x3 + y3
=    + 3x2y – 3xy2 – 6yx2 + 6y3
=      = x3 + y3 + 6yx2 + 3y2x – x3 + y3
– 3xy2 – 6yx2 + 6y3
=      3
= 8y = RHS
      Hence, proved. [Note: Also, see Q. 79, page 48]
=       
A 101. If a + b + c = 0, then prove that :
=       .
a4 + b4 + c4 = 2 (b2c2 + c2a2 + a2b2)
Probable Error : Some students may put (2016-7VJIAJM, IKZKJIT; 2015-3O09O4C, V0JGPRT,

S
x3 = u and obtain u2 + 5u + 8. Then, they may BJOWMSA, D1XDZR5; 2014-HRXKKFA)
may say it cannot be factorised.
Sol. (a + b + c) = 0  (a + b + c)2 = 0.

T SE
A/E 98. If find the value of  a2 + b2 + c2 + 2ab + 2bc + 2ca = 0.
 a2 + b2 + c2 = – (2ab + 2bc + 2ca)
using only the positive value of

H RI
Now (a2 + b2 + c2)2 = a4 + b4 + c4 + 2a2b2
(2016-8JHKR2Y; + 2b2c2 + 2c2a2
2015-VOP2W90, WXD7ZVQ, OTT6CUG)  {(a + b + c)2 – 2ab – 2bc – 2ca}2.

IG P
Sol.  = 34 (Given) = a4 + b4 + c4 + 2a2b2 + 2b2c2 + 2c2a2

R R
 {02 – 2ab – 2bc – 2ca}2
  E  =  +2 = a4 + b4 + c4 + 2a2b2 + 2b2c2 + 2c2a2
  
   4a2b2 + 4b2c2 + 4c2a2 + 8ab2c + 8abc2
= 34 + 2 = 36.
PY T
  + 8a2bc = a4 + b4 + c4 + 2a2b2 + 2b2c2 + 2c2a2
    = ± 6.
O N

   a4 + b4 + c4 = 4a2b2 + 4b2c2 + 4c2a2 – 2a2b2


– 2b2c2 – 2c2a2 + 8abc(a + b + c)
C E

Taking  = 6, we get :
 a4 + b4 + c4 = 2a2b2 + 2b2c2 + 2c2a2 + 8abc × 0
    a4 + b4 + c4 = 2a2b2 + 2b2c2 + 2c2a2 + 0
=      
 
© E

  
 a4 + b4 + c4 = 2(a2b2 + b2c2 + c2a2).
= 6 (34 – 1) = 6 × 33 = 198.
BL

Hence, proved.
[Note : Also see Q. 87, page 62]
U 102. Factorise : x12 – y12
A 99. If x = 2 – y, then show that :
(2016-P2N17UR; 2014-M7F1KZH)
x3 + 6xy + y3 – 8 = 0
IA

Sol. (x ) – (y ) = (x6 – y6) (x6 + y6)


6 2 6 2
(2016-7YAIHMD, MAHMAD, AKRYDAM, 3A9E2QY;
2015-28V72B8; 2014-H9UVP54A) = [(x3)2 – (y3)2](x6 + y6)
EL

Sol. x+y = 2. = [(x3 – y3)(x3 + y3)](x6 + y6)


So, (x + y)3 = (2)3. = (x – y)(x2 + y2 + xy)
or x + y + 3x y + 3y2x
3 3 2
= 8 (x3 + y3)(x6 + y6)
R

or x3 + y3 + 3xy(x + y) = 8 = (x – y)(x + y2 + xy)(x + y)


2

or x3 + y3 + 3xy × 2 = 8 [ x + y = 2] (x2 + y2 – xy)(x6 + y6)


or x3 + y3 + 6xy = 8 = (x – y)(x2 + y2 + xy)(x + y)
 x3 + 6xy + y3 – 8 = 0. (x2 + y2 – xy)(x2 + y2)
Hence, the result. (x4 + y4 – x2y2)
A 100. Prove that :
A/E 103. Factorise : (x2 – 3x)2 – 8(x2 – 3x) – 20
(x + y)3 – (x – y)3 – 6y (x2 – y2) = 8y3 (2016-K2SHJ5P; 2014-Q1DE1Y9)
(2016-WYQXG6V, YY8420J, W8GW7QS, HQU7GIX; Sol. We have : (x – 3x)2 – 8(x2 – 3x) – 20
2

2015-3EFMTRG; 2014-HIX7L6J) (x2 – 3x)2 – 10(x2 – 3x) + 2(x2 – 3x) – 20


Sol. LHS = [(x3 + y3 + 3x2y + 3y2x) = [(x2 – 3x)2 – 10(x2 – 3x)] + [2(x2 – 3x) – 20]
– (x3 – y3 – 3x2y + 3xy2)] – 6yx2 + 6y3 = (x2 – 3x)[(x2 – 3x) – 10] + 2[(x2 – 3x) – 10]
= (x2 – 3x – 10)(x2 – 3x + 2) U 106. Factorise : 4y3 – 12y2 – y + 3
= [x2 – 5x + 2x – 10][x2 – 2x – x + 2]
(2016-7VJIAJM, BBLTON; 2015-ZG3KB1S)
= [x(x – 5) + 2(x – 5)][x(x – 2) – 1(x – 2)] Sol. 4y – 12y2 – y + 3 =
3
4y2 (y – 3) – 1 (y – 3)
= [(x + 2)(x – 5)][(x – 1)(x – 2)]. = (y – 3) (4y2 – 1)
A/E 104. If 3a – 2b + 5c = 5 and 6ab + 10bc – 15ac = (y – 3) {(2y)2 – 12}
= 14, find the value of 27a3 + 125c3 + 90abc – 8b3. = (y – 3) (2y + 1) (2y – 1).
(2016-YTQODJ3, ADBAPR, 3S41HD1, DAVVIJ;
2015-L78MJIK; 2014-RCBFSOJ)
A 107. Simplify :
Sol. Using identity x3 + y3 + z3 – 3xyz     
= (x + y + z)(x2 + y2 + z2 – xy – yz – zx),

S
    
we have : (2016-S16MA11, EGSVY52, HKJJJBK;

T SE
For 3a – 2b + 5c = 5 : 2015-1I03XZ9, 47TTMHC;
Let x = 3a, z = 5c and y = – 2b. 2013-BVNPV, M8XY4C1, 4Z9XL1D;
Putting the values into the identity, we get 2012-43, 49, 52; 2011-460023)
(3a)3 + (–2b)3 + (5c)3 – 3 × (3a) × (–2b) × (5c) Sol. Here, (a2 – b2)  (b2 – c2)  (c2 – a2)  0.

H RI
= [(3a + (–2b) + 5c] [(3a)2 + (–2b)2 + (5c)2 –  (a2 – b2)3  (b2 – c2)3  (c2 – a2)3
3a × (–2b) – (–2b) × (5c) – (5c)(3a)]
 3 (a2 – b2) (b2 – c2) (c2 – a2)

IG P
27a3 – 8b3 + 125c3 + 90abc
 3 (a – b) (a  b) (b – c) (b  c) (c – a) (c  a)
= [3a – 2b + 5c] [9a2 + 4b2 + 25c2 + 6ab

R R
...(1)
+ 10bc – 15ac].
Similarly, (a – b)  (b – c)  (c – a) 0.
We need to find the value of LHS. Let’s solve
RHS.
E  (a – b)3  (b – c)3  (c – a)3
 3(a – b) (b – c) (c – a) ...(2)
PY T
RHS = (3a – 2b + 5c)[9a2 + 4b2 + 25c2 + 6ab
+ 10bc – 15ac]     
O N

2 2 2

= 5[(9a + 4b + 25c ) + 14] ...(1)     
We have to find the value of 9a2 + 4b2 + 25c2.
C E

     

Squaring both sides of 3a – 2b + 5c = 5, we will   
get [From (1) and (2)]
© E

(3a – 2b + 5c)2 = (5)2.  (a  b) (b  c) (c  a).


BL

 (3a)2 + (–2b)2 + (5c)2 + 2 × (3a) × (–2b) + Probable Error : May wrongly obtain the
2 × (–2b) × (5c) + 2 × (3a) × (5c) = 25 answer as (a – b) (b – c) (c – a).
 9a + 4b2 + 25c2 – 12ab + 20bc – 30ac = 25
2 U 108. If (x  y  z)  0, then prove that
IA

 9a2 + 4b2 + 25c2 – 2(6ab – 10bc + 15ac) = 25 (x3  y3  z3 )  3xyz.


 9a2 + 4b2 + 25c2 – 2 × 14 = 25 (2016-NALMAD; 2015-J1Q092O; 2014-H9UVP54A;
2013-TZWM1AM, ULMRL9G; 2011-460013, 27, 38)
EL

 9a2 + 4b2 + 25c2 – 28 = 25


Sol.      
 9a2 + 4b2 + 25c2 = 53 ...(2)
   
Putting this value from eqn. (2) into eqn. (1),

R

we get    
    
RHS = 5[(53) + 14] = 5[67] = 335.
 
Hence, the required value of LHS is 335.
   
A/E 105. Simplify :
    .
(6m – n) (36m2 + 6mn + n2) + (3m + n)3
(2016-N6IZDOQ; 2015-YYH8ND8) Hence,        .
2 2
Sol. (6m – n) (36m + 6mn + n ) + (3m + n) 3 Thus, proved.
= (6m)3 – n3 + (3m + n)3 A 109. Find the value of
= 216m3 – n3 + (3m)3 + 3 (3m)2 n + 3 (3m) n2 + n3
when
= 216m3 – n3 + 27m3 + 27m2n + 9mn2 + n3
= 243m3 + 27m2n + 9mn2. (2016-WYQXG6V, 3WNBW64, IKZKJIT; 2015-BZ27KP0)
Sol. We have :
IV. Assertion-Reason Type
s=  Following questions consist of two statements
Assertion (A) and Reason (R). Answer these
   =0 ...(1) questions, selecting an appropriate option given
below:
Now,   . (a) Both (A) and (R) are true and (R) is the
correct explanation of (A).
(b) Both (A) and (R) are true but (R) is not the
   
=          . correct explanation of (A).
    (c) (A) is true but (R) is false.

S
    (d) (R) is true but (A) is false.
=          
    A/E 7. (A): If a2 + b2 + c2 = 14 and a + b + c = 4,

T SE
then value of ab + bc + ca is equal to 1.
 
     (R): Identity is: (a + b + c)2 = a2 + b2 + c2 +
  2 (ab + bc + ca).

H RI
 
=        SHORT ANSWER TYPE QUESTIONS - I
  A 8. If x = 2y + 6, find the value of x3 – 8y3 –

IG P
= 0. 36xy – 216. (2016-3S41HD1, RRQOKMY)
U 9. Write (3x + 1)3 in the expanded form.

R R 10. Use suitable identity to find (3x + 1) (3x + 1).


U
OBJECTIVE TYPE QUESTIONS
E (2015-CHBCHA)
I. Multiple Choice Questions
11. Factorise :
PY T
U  (2015-HKVECYX)
R 1. Product of      
        
O N

      U 12. Factorise : 6x2 – x – 1 (2015-QJ5TFWP)


is :
C E

SHORT ANSWER TYPE QUESTIONS - II


(A)  (B)  
U 13. If x2 + y2 = 29 and xy = 10, then find the
© E

value of x3 – y3, when x > y.


(C)  (D)  
(2016-SHB8BLL, Z1XF9CV)
BL

(CBSE 2011-460016, 25; 2010-940121-A1, A2) U 14. Prove that (x + y + z) × [(x – y)2 + (y – z)2 +
U 2. (x – y)3 = (z – x)2] = 2 (x3 + y3+ z3 – 3xyz).
IA

(A) x3 + y3 – 3xy(x – y)  
U 15. Simplify by factorisation
(B) x3 – y3 + 3xy(x – y) 
(C) x3 – y3 – 3xy method.
EL

(D) x3 – y3 – 3xy(x – y) A/E 16. If   find the value of 


U 3. 13 + x3 / (1 + x) is
R

(A) 1 + x + x2 (B) 1 + x2 taking only the positive value of 


(C) 1 – x2 (D) 1 – x + x2 (2018-KVS; 2016-KXZ1WCH, 4YWTXY8, LIFRAV;
II. Fill in the blanks 2015-40ZL433)
A 17. Without actually multiplying, find the
1 2 1
A 4. If x   2 , the value of x  2 is ____ . volume of the cuboid, using identities when
x x
L = (x + y) units, B = (x 2 + y 2 – xy) units and
A 5. On factorisation, x2 – y2 – x – y is equal H = (x3 – y3) units. (2018-DoEm)
______. A 18. Simplify :

III. State True or False     


454  441  454  21 21  
A 6. 2 2 is equal to . (2016-WYQXG6V, MN1GSYE; 2015-TYYZTX8)
(877)  (423) 65
 
A 34. If the volume of a cuboid is given by p(x)
A 19. If then find the value of
= (25x2 – 1) + (1 + 5x)2, then find the possible
dimensions. (2018-KVS)
 (2016-1MFC8M5, MN1GSYE, N9O6S82)
LONG ANSWER TYPE QUESTIONS
A/E 20. Prove that : U 35. If a2 + b2 + c2 = 90 and a + b + c = 20,
(a + b)3 + (b + c)3 + (c + a)3 – 3(a + b)(b + c) then find the value of ab + bc + ca.
U 36. Factorise : 27a3 + 8b3 – 18a2b – 12ab2
(c + a) = 2(a3 + b3 + c3 – 3abc). (2018-DoEm)
2 2 (2016-BKV08FS, ETGKH87)
A 21. If x + y = 68 and x – y = 6, then find
A 37. If    and a + b + c = 0,
the value of x3 – y3. (2016-7VJIAJM, 8JHKR2Y)

S
prove that :
A 22. If 2x + 3y = 12 and xy = 6, find the value

T SE
  
of 8x3 + 27y3. (2016-DX82UYF, UGISTKZ)   
   (2016-1MFC8M5; 2014-FWEZ78G;
U 23. Find the product of       2013-KVLUK; 2012-64)
  

H RI
  
     using a suitable identity. A/E 38. If   find the value of 
  

IG P
(2016-9NVBEAE)
A 24. Using a suitable identity, find (98)3. by taking only the positive values of 

R R
(2016-6OGFTVE, DE1F4MQ)
A 25. What are the possible expressions for the
E  etc.
dimensions of a cuboid whose volume is
24ky2 – 4ky – 8k ?
PY T
39. If   find the value of
(2015-A7KIRYZ; 2014-SBADAHQ)
A
  
 
O N

A/E 26. If x and y are two positive real numbers


such that x > 3y, x2 + 9y2 = 369 and xy = 60, find  using only the positive value of 
C E

the value of x – 3y.


(2016-IKZKJIT; 2015-B0WJG93)
(2016-HEFS8NN; 2014-6KH67ZO)
A 40. If a + b + c = 0, then prove that :
© E

A 27. If    = 9, then find the value of



    
BL

   (2018-DoEe)
 .
A 41. Simplify :
U 28. If x + y + z = 12 and x2 + y2 + z2 = 64, then
IA

find xy + yz + zx. (2016-W8GW7QS, D88XL26,


    
K25SHJ5D; 2015-4TBQ2HR) (2016-60GFTVE; 2014-EY48ND6)
EL

U 4
29. Factorise : 9x + 26x – 3 2 U 42. Find the product of

A 30. Find the value of         
          
R

    
(2015-IRUZPDV; 2014-OIJI54C)
A 31. Factorise : by using suitable identity. (2018-KVS)

     A 43. Using a suitable identity factorize :


(2015-9571BXZ; 2014-Q7KDBCR) p3(q – r)3 + q3(r – p)3 + r3(p – q)3 (2018-KVS)
 
U 32. Expand    
 
(2016-ETGKH87, OU6BHOO, L1I5XFF; 1. (C) 2. (B)
2015-GOVDEM) 3. (D) 4. 2
A 33. Without actually calculating the cubes,
5. (x + y) (x – y – 1) 6. True
find the value of (– 12)3 + (7)3 + (5)3.
(2018-DoEm) 7. (a) 8. 0
9. 27x3 + 27x2 + 9x + 1 10. 9x2 + 6x + 1 26. 3 27. 702 28. 40
29. (x2 + 3) (3x – 1) (3x + 1) 30. 60
  
11.      12. (2x – 1) (3x + 1) 31. 3(x – 2y)(2y – 3z)(3z – x)
  
 32.     
13. 117 15.

33. –1260 34. 10, x, 5x + 1 35. 155
16. 18 [Probable Error : May also obtain the value – 18 36. (3a + 2b) (3a – 2b) (3a – 2b)

38. 39. 76
using  = – 3]

S
17. x6 – y6 18. – 166 3 19. 322 41. –19(3x3 + y3) 42.

21. 504 22. 432 23. a8 – 43. 3pqr(p – q)(q – r)(r – p) [Check : May obtain 3pqr

T SE
(p + q) (q + r) (r + p)
24. 941192 25. 4k, (2y + 1) and (3y – 2)

H RI
1. Teacher wrote of polynomial x2 + kx + 8 on the (e) What will be the remainder, if the

IG P
U
blackboard and said that value of this polynomial at polynomial is divided by (x + 5)?

R R
x = –3 is –1. (i) – 87 (ii) – 89 (iii) – 91 (iv) – 93
E Answers: (a) (i) (b) Cubic (c) (ii)
(d) (iv) (e) (ii)
A 3. Smriti went to a orphanage and gave a cuboidal
PY T
gift whose volume was given by a polynomial
(a) Value of k is 4x3 + x2 – 11x + 2r.
O N

(i) 6 (ii) –6 (iii) 9 (iv) –9 (a) If length of the cuboid is (4x – 3), then
C E

(b) What type of polynomial is it? value of r is


(c) Zeroes of this polynomial are (i) 11 (ii) 4 (iii) 3 (iv) 2
(i) 4 and 2 (ii) 4 and –2 (b) Degree of the polynomial is
© E

(iii) –4 and 2 (iv) –4 and –2 (i) 4 (ii) 3 (iii) 2 (iv) 1


BL

(d) Value of the polynomial at x = 3 is (c) What could be the breadth and height of
(i) 1 (ii) 18 (iii) 25 (iv) 35 the cuboid?
(e) Value of the polynomial at x = –5 is (i) (x + 2) and (x – 1)
IA

(i) –3 (ii) 3 (iii) –7 (iv) 7 (ii) (x – 2) and (x + 1)


Answers: (a) (i) (b) Quadratic (c) (iv) (iii) (x + 2) and (x + 1)
(iv) (x – 2) and (x – 1)
EL

(d) (iv) (e) (ii)


A 2. Teacher asked the students to state the value (d) If x = 3, then dimensions of the cuboid
will be
of ‘a’, if on dividing the polynomial y 3 – 4y + a by
(i) 9, 1 and 4 (ii) 9, 5 and 2
R

(y – 3), remainder is 31.


(iii) 9, 5 and 4 (iv) 9, 1 and 2
(a) Which of the following responses of the
students is correct? (e) Cost of the gift given by Smriti at the rate
of 5 per cubic units will be
(i) 16 (ii) 13 (iii) –13 (iv) –16
(i) 180 (ii) 90 (iii) 900 (iv) 450
(b) What type of a polynomial is it?
Answers: (a) (iii) (b) (ii) (c) (i)
(c) What will be the remainder, if the (d) (ii) (e) (iv)
polynomial is divided by (x + 2)?
A 4. Three friends A, B and C of locality decided to
(i) 32 (ii) 16 (iii) 12 (iv) 8
start a business with a capital represented by a
(d) What will be remainder if the polynomial polynomial x 3 + kx 2 – x + 5, which is the product of
is divided by (x – 5)? their shares, such that shares of A, B and C are in
(i) 115 (ii) 117 (iii) 119 (iv) 121 decreasing order.
(a) If share of A in the capital polynomial is are in the form of irreducible factors of the total
(x + 1), then value of k is contribution.
(i) 1 (ii) –1 (iii) 5 (iv) –5 (a) How you will find the contribution of each
(b) The polynomial is a student?
(i) constant polynomial (b) Which algebraic identities will help you in
(ii) linear polynomial the process?
(iii) quadratic polynomial (c) Factorisation of x12 – y12 is
(iv) cubic polynomial (i) (x – y) (x + y) (x2 + y2 + xy) (x4 + y4 – x2y2)
(c) Share of B in the capital is (ii) (x – y) (x2 + y2 – xy) (x4 + y4 – x2y2)
(i) (x + 1) (ii) (x – 1) (iii) (x + y) (x – y) (x2 + y2) (x2 + y2 – xy) (x2 +

S
(iii) (x – 2) (iv) (x – 5) y2 + xy) (x4 + y4 – x2y2)
(d) Share of C in the capital is (iv) (x + y) (x – y) (x2 + y2) (x2 + y2 – xy) (x4 +

T SE
y + x2y2)
4
(i) (x – 1) (ii) (x – 3)
(d) Number of students, who contributed the
(iii) (x – 5) (iv) (x + 5)
money, is
(e) If x = 20, then total capital (in ) is

H RI
(i) 3 (ii) 4 (iii) 5 (iv) 6
(i) 10025 (ii) 10005
(e) If x = 2 and y = 1, then the total
(iii) 9995 (iv) 9985
contribution (in ) is

IG P
Answers: (a) (iv) (b) (iv) (c) (ii)
(i) 4195 (ii) 4095
(d) (iii) (e) (iv)
(iii) 4083 (iv) 3993

R R
A/E 5. Some of the students of class IX contributed Answers: (a) By factorising x12 – y12
some money to help in the education of needy
E (b) a2 – b2 = (a + b) (a – b), a3 + b3 = (a + b)
students. Their total contribution is in the form of a
(a – ab + b2) and a3 – b3 = (a – b) (a2 + ab + b2)
2
polynomial x12 – y12, which is the product of the
PY T
(c) (iii) (d) (iv) (e) (ii)
contributions of all the students. These contributions
O N
C E

Time Allowed : 1:30 hours Maximum Marks : 40


© E

General instructions :
(i) Do all the questions given in the chapter test.
BL

(ii) Questions from 1 to 10 carry 1 mark each.


(iii) Questions from 11 to 13 carry 2 marks each.
(iv) Questions from 14 to 17 carry 3 marks each.
IA

(v) Questions from 18 to 20 carry 4 marks each.


(vi) Get the test paper checked by your teacher.
EL

Multiple Choice Question


R 1. Which of the following is a cubic polynomial ? (1)
R

3 2 2 2 2
(A) x  3x – 4x  3 (B) x  4x – 7 (C) 3x  4 (D) 3(x  x  1)
Fill in the blanks
R 2. x2 + 5x + 4 is a ______ polynomial. (1)
R 3. If polynomial p(x) is divided by (x + 3), then the remainder is ______. (1)

State True or False


U 4. (x – 1) is a factor of x2 + x – 2. (1)
R 5. If (x + y + z) = 0, then x3 + y3 + z3 is equal to xyz2. (1)

Assertion-Reason Type
Following questions consist of two statements Assertion (A) and Reason (R). Answer these questions, selecting
appropriate option given below:
(a) Both (A) and (R) are true and (R) is the correct explanation of (A).
(b) Both (A) and (R) are true but (R) is not the correct explanation of (A).
(c) (A) is true but (R) is false.
(d) (R) is true but (A) is false.
A 6. (A): (x + 2) is a factor of 4x3 + 13x + 58. (1)
(R): 4(–2)3 + 13(–2) + 58 = 0.
A/E 7. (A): Value of 303 – 483 + 183 is equal to –77760. (1)
(R): 30 – 48 + 18 = 0, 50, 303 – 483 + 183 = –3 × 30 × 48 × 18.
Match the columns

S
A/E 8. (1)

T SE
Column-1 Column-2
1. (x – 1) is a factor of (a) 2x2 – x – 15
2. (x + 2) is a factor of (b) x140 + 2x139 + 1

H RI
3. (x – 3) is a factor of (c) x2 + x – 6
4. (x + 1) is a factor of (d) x2 + 2x – 3
(e) 4x3 – 13x + 6

IG P
5. (x – 2) is a factor of

9. Find the value of p for which x  p is a factor of x2  px  3 – p. (1)

R R
U
3 3 3
A 10. The factorisation of (2a – b)  (b – 2c)  8(c – a) is :
E (1)
3 2
A 11. If y = 2 and y = 0 are the zeroes of the polynomial f (y) = 2y – 5y + ay + b, find the values of
a and b. (2)
PY T
A 12. Using remainder theorem, check whether the polynomial 2x3 – 2ax2 – 6x + 6a is a multiple of
O N

x – a. (2)
2 
A/E 13. If x + a is a factor of ma – nx – 3x , then prove that  (2)
C E

A/E 14. Prove that 9x2 + 30x + 28 has no zeroes. (3)


© E

   
U 15. Simplify :      (3)
   
BL

A 16. Factorise :    (3)

U 17. If a2 + b2 + c2 = 80 and a + b + c = 20, then find the value of ab + bc + ca. (3)


IA

4 3
A 18. Divide the polynomial 3x + 4x – 4x +3 by x – 1 and verify the remainder by using remainder
theorem. (4)
EL

A/E 19. Find what must be subtracted from the polynomial 4y4 – 12y3 + 6y2 + 50y + 26 so that the
obtained polynomial is exactly divisible by y2 + 4y + 4. (4)
A/E 20. If      is divided by (x – 1) and (x + 1), it leaves the remainders 5
R

and 19 respectively. Find the remainder when f (x) is divided by (x – 2). (4)

1. (A) 2. quadratic 3. p(–3) 4. True 5. False 6. (a) 7. (a)

8. 1 (d); 2 (e); 3 (a); 4 (b); 5 (c) 9. 3 10. 6(2a – b)(b – 2c)(c – a)

   
11. a = 2, b = 0 12. Yes 15.  16.      
  
17. 160 18. Remainder = 6 19. 110 20. 10
SAMPLE
QUESTI N PAPER
MATHEMATICS (TERM-II)
1
Time allowed : 90 Minutes CLASS–IX Maximum Marks : 40
General Instructions :

S
1. The question paper contains three parts A, B and C.
2. Section – A consists of 20 questions of 1 mark each. Any 16 questions are to be attempted.

T SE
3. Section – B consists of 20 questions of 1 mark each. Any 16 questions are to be attempted.
4. Section – C consists of 10 questions based on two Case Studies. Attempt any 8 questions.
5. There is no negative marking.

H RI
Section – A
(Section–A consists of 20 questions of 1 mark each. Any 16 questions are to be attempted.)
Multiple Choice Questions

IG P
1. Factors of x4 – x2 – 12 are

R R
(A) (x + 2), (x2 – 2), (x2 + 3) (B) (x + 3), (x – 3), (x2 + 2)
(C) (x + 2), (x – 2), (x2 – 3)
E (D) (x2 + 2), (x2 – 6)
2
 1 1
2. If  x –   x2  y  2 , then the value of y is
PY T
 x x
O N

(A) –2 (B) 2 (C) 2x (D) –2x


3. x2 + (a + b + c)x + ab + bc =
C E

(A) (x + a)(x + b + c) (B) (x + a)(x + a + c) (C) (x + b)(x + a + c) (D) (x + b)(x + b + c)


4. Factorisation of a2x – b2x is
© E

(A) (ax + bx)(ax – bx) (B) (ax – bx)2 (C) (ax + bx)(ax – b2) (D) (ax – bx)(ax + b2)
5. Which one of the following algebraic expressions is a polynomial in variable x ?
BL

3/2
2 1 2 3x
(A) x2  (B) x (C) x  (D) None of these
x2 x x
IA

6. If one factor of 5 + 8x – 4x2 is (2x + 1), then the second factor is


(A) (5 + 2x) (B) (2x – 5) (C) (5 – 2x) (D) – (5 + 2x)
7. The two diagonals are equal in a
EL

(A) parallelogram (B) rhombus (C) rectangle (D) trapezium


8. A quadrilateral has three acute angles each measuring 70°. The measure of fourth angle is
(A) 140° (B) 150° (C) 105° (D) 120°
R

9. In a cyclic quadrilateral, the difference between two opposite angles is 58°, the measures of opposite
angles are
(A) 158°, 22° (B) 129°, 51° (C) 109°, 71° (D) 119°, 61°
10. Which of the following statements is true for a regular pentagon ?
(A) All vertices are con-cyclic. (B) All vertices are not con-cyclic.
(C) Only four vertices are con-cyclic (D) Cannot say anything about regular pentagon
11. Which of the following angles can be constructed by using ruler and compass only ?
(A) 20° (B) 72° (C) 105° (D) 130°
12. The construction of a LMN in which LM = 8 cm, L = 45° is possible when (MN + LN) is
(A) 6 cm (B) 7 cm (C) 9 cm (D) 5 cm
13. Vertical cross-section of a right circular cylinder is always a
(A) rectangle (B) rhombus (C) square (D) trapezium
Assertion/Reason
In each of the following questions, a statement of Assertion is given followed by a
corresponding statement of Reason just below it. Of the statements, mark the correct answer
as
(A) Both Assertion and Reason are true and Reason is the correct explanation of Assertion.
(B) Both Assertion and Reason are true but Reason is not the correct explanation of Assertion.
(C) Assertion is true but Reason is false.
(D) Assertion is false but Reason is true.
14. Assertion : If f(x) = 3x7 – 4x6 + x + 9 is a polynomial, then its degree is 7.
Reason : Degree of a polynomial is the highest power of the variable in it.
15. Assertion : Two opposite angles of a parallelogram are (3x – 2°) and (50 – x)°. The measure of one

S
of the angle is 37°.
Reason : Opposite angles of a parallelogram are equal.

T SE
16. Assertion : The length of a chord which is at a distance of 5 cm from the centre of a circle of radius
10 cm is 17.32 cm.
Reason : The perpendicular from the centre of a circle to a chord bisects the chord.

H RI
17. Assertion : The circumference of a circle must be a positive real number.
Reason : If r(> 0) is the radius of the circle, then its circumference 2 r is a positive real number.
18. Assertion : The external dimensions of a wooden box are 18 cm, 10 cm and 6 cm respectively and

IG P
thickness of the wood is 15 mm, then the internal volume is 765 cm 3.
Reason : If external dimensions of a rectangular box be l, b and h and the thickness of its sides

R R
be x, then its internal volume is (l – 2x)(b – 2x)(h – 2x).
Match the Column E
In the section, each question has two matching lists. Choices for the correct combination
PY T
of elements from Column-I and Column-II are given as options (A), (B), (C) and (D) out of which
one is correct.
O N

19. Column-II shows the degree of polynomials given Column-I


Column-I Column-II
C E

(P) 2 – y2 – y3 + 2y8 (1) 2


(Q) 2 (2) 1
© E

(R) 5x – 7 (3) 0
BL

(S) 4 – x2 (4) 8
(A) P 1, Q 3, R 2, S 4 (B) P 4, Q 3, R 2, S 1
(C) P 4, Q 2, R 3, S 1 (D) P 2, Q 3, R 4, S 1
IA

20. Match the following :


Column-I Column-II
EL

(P) Trapezium (1) Each angle is 90°


(Q) Rectangle (2) Equal adjacent sides but unequal opposite sides.
(R) Rhombus (3) Unequal sides.
R

(S) Kite (4) All sides are equal.


(A) P 1, Q 2, R 3, S 4 (B) P 2, Q 3, R 4, S 1
(C) P 4, Q 3, R 2, S 1 (D) P 3, Q 1, R 4, S 2

Section – B
(Section–B consists of 20 questions of 1 mark each. Any 16 questions are to be attempted.)
Multiple Choice Questions
21. Degree of the polynomial p(x)= 3x4 + 6x + 7 is
(A) 4 (B) 5 (C) 3 (D) 1
22. If one factor of a(x + y + z) + bx + by + bz is (x + y + z), then the second factor is
(A) ax + ay + az (B) bx + by + bz (C) bx + by – bz (D) a + b
23. Degree of the polynomial p(x) = (x + 2)(x – 2) is
(A) 2 (B) 1 (C) 0 (D) 3
24. If 8x4 – 8x2 + 7 is divided by 2x + 1, the remainder is
11 13 15 17
(A) (B) (C) (D)
2 2 2 2
25. 3 3
If a + b + c = 0, then a + b + c = 3

(A) abc (B) 3abc (C) 2abc (D) – 3abc


26. For the polynomial p(x) = x5 + 4x3 – 5x2 + x – 1, one of the factors is
(A) (x + 1) (B) (x – 1) (C) x (D) (x + 2)
27. A quadrilateral having only one pair of opposite sides parallel is called a
(A) square (B) rhombus (C) trapezium (D) parallelogram
28. Two adjacent angles of a parallelogram are (2x + 25°) and (3x – 5)°. The value of x is

S
(A) 28° (B) 32° (C) 36° (D) 42°
29. In a cyclic quadrilateral ABCD, if two sides are parallel, which of the following statements is definitely

T SE
false ?
(A) Remaining two sides are equal (B) Diagonals are not equal
(C) Diagonals intersect at the centre of circle (D) Both (A) and (C)
30. To construct a right triangle whose base is 12 cm and sum of its hypotenuse and other side is 18 cm.

H RI
We draw line segment AB of 12 cm. Draw a ray AX making 90° with AB. The next step is :
(A) Cut a line segment AD of 18 cm on AX (B) Cut a line segment BD of 18 cm
(C) Cut a line segment AD of 18 cm on AB (D) Cut a line segment BD of 18 cm on AB

IG P
31. The ratio of the volume and surface area of a sphere of unit radius without considering units is

R R
(A) 4 : 3 (B) 3 : 4 (C) 1 : 3 (D) 3 : 1
32. The number of surfaces in right cylinder is

33.
(A) 4
E (B) 3 (C) 2 (D) 1
A rectangular sheet of metal, x cm by y cm has a square of size z cm cut from each corner. The sheet
PY T
is then bent to form a tray of depth z cm. The volume of the tray is
(A) z(x – z) (y – z) cm3 (B) xyz cm3
O N

(C) z(x – 2z) (y – 2z) cm 3 (D) (x + y)z cm3


C E

Assertion/Reason
In each of the following questions, a statement of Assertion is given followed by a
corresponding statement of Reason just below it. Of the statements, mark the correct answer
© E

as
(A) Both Assertion and Reason are true and Reason is the correct explanation of Assertion.
BL

(B) Both Assertion and Reason are true but Reason is not the correct explanation of Assertion.
(C) Assertion is true but Reason is false.
(D) Assertion is false but Reason is true.
IA

34. Assertion : (x + 2) and (x – 1) are factors of the polynomial x4 + x3 + 2x2 + 4x – 8.


Reason : For a polynomial p(x) of degree 1, x – a is a factor of the polynomial p(x) if and only if
EL

p(A) 1.
35. Assertion : The angles of a quadrilateral are x°, (x – 10)°, (x + 30)° and (2x)°, the smallest angle
is equal to 58°.
Reason : Sum of the angles of a quadrilateral is 360°.
R

36. Assertion : If the diagonals of a parallelogram ABCD are equal, then ABC = 90°.
Reason : If the diagonals of a parallelogram are equal, it becomes a rectangle.
37. Assertion : The measure of AOC = 60°.

Reason : Angle subtended by an arc of a circle at the centre of the circle is double the angle
subtended by arc on the circumference.
38. Assertion : A cone is a solid figure.
Reason : A cone is generated when rectangular sheet is rotated about its axis.
Match the Column
In the section, each question has two matching lists. Choices for the correct combination
of elements from Column-I and Column-II are given as options (A), (B), (C) and (D) out of which
one is correct.
39. By using a given figure of quadrilateral ABCD, match Column-I with Column-II.

Column-I Column-II

S
(P) If ABCD is a parallelogram, then sum of the angles x, y and z is (1) 25°
(Q) If ABCD is a rhombus, where D = 130°, then the value of x is (2) 180°

T SE
(R) If ABCD is a rhombus, the value of w is (3) 50°
(S) If ABCD is a parallelogram, where x + y = 130°, the value of z is (4) 90°
(A) P 1, Q 2, R 3, S 4 (B) P 3, Q 4, R 2, S 1

H RI
(C) P 2, Q 1, R 4, S 3 (D) P 2, Q 4, R 3, S 1
40. Match the following :

IG P
Column-I Column-II
(P) The radius of circle is 8 cm and the length of one of its chords is 12 cm. (1) 23 cm

R R
The distance of the chord from the centre is
(Q) Two parallel chords of lengths 30 cm and 16 cm are drawn on the (2) 5.196 cm
E
opposite sides of the centre of a circle of radius 17 cm. The distance
PY T
between the chords is
(R) The length of a chord which is at a distance of 4 cm from the centre (3) 5.291 cm
O N

of the circle of radius 6 cm is


(S) An equilateral triangle of side 9 cm is inscribed in a circle. (4) 8.94 cm
C E

The radius of the circle is


(A) P 3, Q 1, R 4, S 2 (B) P 3, Q 4, R 1, S 2
© E

(C) P 1, Q 2, R 3, S 4 (D) P 1, Q 3, R 2, S 4
BL

Section – C
(Section C consists of 10 questions of 1 mark each. Any 8 questions are to be attempted.)
Case Study Based Questions
IA

CASE STUDY-1 : Mathematics teacher of a school


took her 9th standard students to show Red fort. It
was a part of their Educational trip. The teacher had
EL

interest in history as well. She narrated the facts of


Red fort to students. Then the teacher said in this
monument one can find combination of solid figures.
R

There are 2 pillars which are cylindrical in shape.


Also 2 domes at the corners which are hemispherical.
7 smaller domes at the centre. Flag hoisting ceremony
on Independence Day takes place near these domes.
41. How much cloth material will be required to cover 2 big domes each of radius 2.5 metres ?
(Take = 22/7)
(A) 75 m2 (B) 78.57 m2 (C) 87.47 m2 (D) 25.8 m2
42. Write the formula to find the volume of a cylindrical pillar :
(A) r2h (B) rl (C) l (l + r) (D) 2 r
43. Find the lateral surface area of two pillars if height of the pillar is 7 m and radius of the base is 1.4 m.
(A) 112.3 cm2 (B) 123.2 m2 (C) 90 m2 (D) 345.2 cm2
44. How much is the volume of a hemisphere if the radius of the base is 3.5 m ?
(A) 85.9 m3 (B) 80 m3 (C) 98 m3 (D) 89.83 m3
45. What is the ratio of sum of volumes of two hemispheres of radius 1 cm each to the volume of a sphere
of radius 2 cm ?
(A) 1 : 1 (B) 1 : 8 (C) 8 : 1 (D) 1 : 16
CASE STUDY-2 : Aditi runs a handicraft shop in
Bapu bazar in Jaipur. She makes beautiful necklaces
using colourful beads which she keeps in a potli. Today
she prepared 19 necklaces but could not make the 20th
necklace as she had no yellow beads left. She counted

S
the beads and found that there were 8 red, 6 green and
14 blue beads remaining in her potli. Her little daughter

T SE
Dulari requested for a bead. Aditi decides to take out
one bead from her potli for Dulari.
46. Find the probability that she draws a green bead.
(A) 3/11 (B) 3/7 (C) 11/14 (D) 3/14

H RI
47. Find the probability that the bead drawn by her is not green.
(A) 3/11 (B) 3/7 (C) 11/14 (D) 3/14

IG P
48. Find the probability that she draws either a green or a blue bead.
(A) 5/7 (B) 5/12 (C) 7/12 (D) 3/14

R R
49. Find the probability that she draws neither a red nor a green bead.
(A) 3/14 E (B) 1/3 (C) 3/7 (D) 1/2
50. Which of the following is an impossible event?
(A) The bead drawn is not red
PY T
(B) The bead drawn is neither red nor blue
(C) The bead drawn is either red or green or blue.
O N

(D) The bead drawn is yellow.


C E

1. (A) (x + 2), (x2 – 2), (x2 + 3) 2. (A) –2 3. (C) (x + b)(x + a + c)


© E

3/2
3x
4. (A) (ax + bx)(ax – bx) 5. (C) x2  6. (C) (5 – 2x)
BL

x
7. (C) rectangle 8. (B) 150° 9. (D) 119°, 61°
10. (A) All vertices are con-cyclic. 11. (C) 105° 12. (C) 9 cm
IA

13. (A) rectangle 14. (A) 15. (A)


16. (A) 17. (A) 18. (A)
19. (B) P 4, Q 3, R 2, S 1 20. (D) P 3, Q 1, R 4, S 2
EL

21. (A) 4 22. (D) a + b 23. (A) 2


11
24. (A) 25. (B) 3abc 26. (B) (x – 1)
2
R

27. (C) trapezium 28. (B) 32°


29. (B) Diagonals are not equal 30. (A) Cut a line segment AD of 18 cm on AX
31. (C) 1 : 3 32. (B) 3 33. (C) z(x – 2z) (y – 2z) cm3
34. (C) 35. (A) 36. (A)
37. (D) 38. (C)
39. (C) P 2, Q 1, R 4, S 3 40. (A) P 3, Q 1, R 4, S 2
41. (B) 78.57 m 2 42. (A) 2
r h 43. (B) 123.2 m2
44. (D) 89.83 m3 45. (B) 1 : 8 46. (D) 3/14
47. (C) 11/14 48. (A) 5/7 49. (D) 1/2
50. (D) The bead drawn is yellow.
SAMPLE
QUESTI N PAPER
MATHEMATICS (TERM-II)
4
Time allowed : 90 Minutes CLASS–IX Maximum Marks : 40
General Instructions :

S
1. The question paper comprises two parats I and II. There are 29 questions in the question paper. All questions are
compulsory.

T SE
2. Part – I : Question no. 1 to 20 all questions and parts thereof are of one mark each. These questions contain Multiple
Choice Questions (MCQs) and Case Study Based Questions. Answers to these should be given in correct options.
3. Part – II : Section–A : Question no. 21 to 23 are Short Answer Type Questions, carrying 2 marks each. Student has
to attempt any two questions in these questions.

H RI
4. Section–B : Question no. 24 to 26 are Short Answer Type Questions, carrying 3 marks each. Student has to attempt
any two questions in these questions.
5. Section–C : Question no. 27 to 29 are Long Answer Type Questions, carrying 5 marks each. Student has to attempt

IG P
any two questions in these questions.
6. Wherever necessary, neat and properly labelled diagrams should be drawn.

R RE Part – I
(This part consists of 20 questions of 1 mark each.)
PY T
Multiple Choice Questions
O N

1. If x2  kx  6  (x  2)(x  3) for all x, the value of k is :


(A) 1 (B) – 1 (C) 5 (D) 3
C E

2. D, E and F are mid-points of sides BC, CA and AB of ABC. If perimeter of ABC is


12.8 cm, then perimeter of DEF is :
© E

(A) 17 cm (B) 38.4 cm (C) 25.6 cm (D) 6.4 cm


3. If PQRS is a parallelogram, then Q – S is equal to :
BL

(A) 90° (B) 120° (C) 180° (D) 0°


4. If in a quadrilateral ABCD, A = 90° and AB = BC = CD = DA, then ABCD is a :
(A) parallelogram (B) rectangle (C) square (D) rhombus
IA

5. Three chords AB, CD and EF of a circle are respectively 3 cm, 3.5 cm, and 3.8 cm away from the
centre. Then which of the following is correct ?
EL

(A) AB > CD > EF (B) AB < CD < EF (C) AB  CD  EF (D) AB  CD < EF


6. AD is a diameter of a circle and AB is a chord. If AD = 34 cm and AB = 30 cm, the distance of AB from the
centre of the circle is :
(A) 17 cm (B) 15 cm (C) 4 cm (D) 8 cm
R

7. The region between an arc and two radii, joining the centre to the end points of the are is called :
(A) Sector (B) Segment (C) Semicircle (D) None of the above
8. The radii of two right circular cylinders are in the ratio of 4 : 5 and their heights are in the ratio 2 :
3. The ratio of their curved surface area is equal to :
(A) 8 : 15 (B) 36 : 81 (C) 2 : 3 (D) 16 : 25
9. The radius of a cylinderical wire is decreased to one-third. If its volume remains the same, its length
will increase to :
(A) 2 times (B) 3 times (C) 6 times (D) 9 times
10. Which of the following cannot be experimental probability of an event ?

(A) (B) 1 (C) 0 (D)


Case Study Based Questions
CASE STUDY-1 : In a newly constructed park which is
situated in the heart of city Hyderabad, an architect has form
a structure in the given shape. The shape has a cuboid, which
is standing on the two cylindrical beams. The dimensions of
the cuboid are 1.5 m, 3 m and 0.5 m. The dimensions of the
cylinders are of height 2 m and diameter 0.6 m.
11. As the structure is made from the concrete, how much
volume of concrete is required to make the cuboidal shape?
(A) 1.75 m3 (B) 2.20 m3
(C) 2.25 m 3 (D) 1.25 m3

S
12. What is formula for calculating the lateral surface area of the cylinder ?

T SE
(A) r2 (B) 2 rh (C) r2h (D) 2 r3
13. What is the volume of two cylinders ?
(A) 1.20 m3 (B) 1.134 m3 (C) 3 m3 (D) 2.2 m3
14. If the cuboid needs to be painted red, how much area need to be painted ?

H RI
(A) 5.2 m2 (B) 13 m2 (C) 6.75 m2 (D) 5.7 m2
15. If a cloth is needed to cover the cylindrical part, how much cloth is needed ?

IG P
(A) 8.25 m2 (B) 1.25 m2 (C) 4.50 m2 (D) 7.536 m2
CASE STUDY-2 : Diwali Fest is an annual South Asian

R R
arts and culture festival, produced by the Diwali Celebration
Society. In the Diwali fest, a game is played which is like
E
that, there is a spinner on which some numbers are written.
The numbers on spinner are 2, 5, 7, 9, 12, 16. Depending
PY T
on the condition of stall owner, if a particular number comes,
O N

than a die will be thrown.


16. What is the probability, that spinner stops on an even number?
C E

1 1 1 1
(A) (B) (C) (D)
8 4 2 16
17. A player will get a special prize, if spinner stops on a perfect square:
© E

1 1 1 1
(A) (B) (C) (D)
BL

3 2 4 8
18. If the player gets a chance to throw, a dice, what is the probability of getting a multiple of 2 on dice?
1 1 1 1
(A) (B) (C) (D)
IA

3 2 4 8
19. If a dice is thrown, what is the probability of getting a number less than 4?
1 1 1 2
EL

(A) (B) (C) (D)


2 4 8 7
20. An event whose probability to occur is 1. Then this type of event is called :
(A) Impossible event (B) Possible event (C) Inependent event (D) Sure event
R

Part – II
Section – A
(This Section consists of 3 questions of 2 mark each. Any 2 questions are to be attempted.)
21. Find the value of the polynomial p( z = 3 z2 – 4 z + 17 , when z  3. A
22. In the given figure, O is the centre of the circle, OM  BC, OL  AB, ON  AC
and OM = ON = OL. Is ABC equilateral ? Give reasons. L
O N
B
23. The height of the cone is 15 cm. If its volume is 1570 cm3, find the radius of the M
C
base (use  = 3.14).
Section – B
(This Section consists of 3 questions of 3 mark each. Any 2 questions are to be attempted.)
–1
24. If x  is a zero of the polynomial p(x)  27x3 – ax2 – x  3, then find the value of a.
3
25. If the polynomial p(x)  x4 – 2x3  3x2 – ax  8 is divided by (x – 2), it leaves a remainder 10. Find the
value of a.
26. 1.1 cu.cm of copper is to be drawn into a cylindrical wire 0.5 cm in diameter. Calculate the length of
the wire.

Section – C
(This Section consists of 3 questions of 5 mark each. Any 2 questions are to be attempted.)

S
27. The polynomials x3  2x2 – 5ax – 8 and x3  ax2 – 12x – 6 when divided by (x – 2) and (x – 3) leave

T SE
remainders p and q respectively. If q – p  10, find the value of a.
28. If X, Y and Z are the mid-points of sides BC, CA and AB of ABC respectively, prove that AZXY is a
parallelogram.
29. In the given alongside figure, ABC is a triangle inscribed in a circle, with centre O, such that AB 

H RI
AC and BEC  100°. Find the value of x and y.

IG P
R RE
PY T
O N

Answers
C E

1. (C) 5 2. (D) 6.4 cm 3. (D) 0° 4. (C) square


5. 6. 7. 8.
© E

(A) AB > CD > EF (D) 8 cm (A) Sector (A) 8 : 15


9. (D) 9 times 10. (D) 11. (C) 2.25 m3 12. (B) 2 rh
BL

1
13. (B) 1.134 m3 14. (B) 13 m2 15. (D) 7.536 m2 16. (C)
2
1 1 1
17. (A) 18. (B) 19. (A) 20. (D) Sure event
IA

3 2 2
21. p(z) 3z2 – 4z 
EL

So, p(3)     1
         1
22. Given : OL  AB, OM  BC and ON  AC ½
R

Also, OM = ON = OL = Perpendicular distance of chords from the centre of a circle ½


 AB = BC = AC [Chords equidistant from the centre of a circle are equal.] ½
ABC is an equilateral triangle. ½
23. Let the radius of the base of the cone be r cm.
Height, h = 15 cm (Given)
3
Volume of cone = 1570 cm (Given)
So,  = 1570 1

or    = 1570

 r2 =  ½

 r = 10 cm. ½
24. If x  – is a zero of the polynomial
27x3 – ax2 – x  3,
     
then          0 1
     
   
or       0 1
   

or     0

S

or   

T SE
 
or 
or a  21 1

H RI
25. p(x)    
Using Remainder Theorem

IG P
10  p(2)         1
or 10  16 – 16  12 – 2a  8 1

R R
or – 2  – 2a  8 ½
or – 2a  – 10
E
i.e., a 5 ½
26. Let the length of wire  l cm ½
PY T
O N

Here ‘r’   0.25 cm ½


C E

 Volume of wire  r2h ½


  (0.25)2 h
But, volume of the copper  1.1 cm3
© E

   
BL

Therefore, [Here h = l] 1

  
 l    5.6 cm ½
IA

27. Let p(x)  x3  2x2 – 5ax – 8 and


g(x)  x3  ax2 – 12x – 6
EL

When divided by (x – 2) and (x – 3) leave remainders p and q respectively


p(x)  x3  2x2 – 5ax – 8
Now, p(2)  23  2 × 22 – 5a × 2 – 8 1½
R

 8  8 – 10a – 8
So, p  8 – 10a ...(1)
Also, g(3)  33  a × 32 – 12 × 3 – 6
So, q  27  9a – 36 – 6 1½
or q  –15  9a
Now, it is given that q – p  10
Therefore, – 15  9a – 8  10a  10 1
 19a – 23  10
 19a  33

 a  1
28. In ABC
X and Z are mid-points of sides BC and AB respectively.
 XZ || AC (By mid point-theorem)
or XZ || AY 1½
X and Y are mid-points of sides BC and AC respectively
 XY || AB (By mid-point theorem) 1½
 XY || AZ
Since in quad AZXY, both pair of opposite sides are parallel
AZXY is parallallelogram. 2
29. ABCD is a cyclic quadrilateral.

S
 A  100°  180° (Opposite angles of cyclic quadrilateral)
 A  80° 1

T SE
In ABC, AB  AC (Given)
 ABC  ACB  x 1
 A  2 x  180° (Angle sum property of a triangle)

H RI
 2x  180° – 80° = 100° 1
 x  50° 1

IG P
BDCE is also a cyclic quadrilateral
 y  100°  180°  y  180° – 100° = 80° 1

R RE
PY T
O N
C E
© E
BL
IA
EL
R
R
EL
I AB
© LE
C E
O N
PY T
R RE
IG P
H RI
T SE
S

You might also like